QP Science Compile

You might also like

Download as pdf or txt
Download as pdf or txt
You are on page 1of 130

कें द्रीय विद्यालय संगठन

(जम्मू संभाग )

KENDRIYA VIDYALAYA SANGATHAN


(JAMMU REGION)

SAMPLE PAPER
FOR CLASS X
SCIENCE (086)

क्षेत्रीय कायाालय, जम्मू, नज़दीक राजकीय विककत्सालय, गााँधी नगर


जम्मू-180004
Regional Office, Jammu, Near Govt. Hospital,
Gandhi Nagar, Jammu-180004
KENDRIYA VIDYALAYA SANGATHAN, JAMMU REGION
SAMPLE PAPER SET 01
CLASS - X SUBJECT : SCIENCE (086)
TIME : 3 hours M. M : 80

General Instructions

I) This question paper consists of 39 Questions in 5 sections.


II) All questions are compulsory. However, an internal choice is provided in some
questions. A student is expected to attempt only one of these questions.
III) Section A consists of 20 objective type questions carrying one marks each.
IV) Section B consists of 6 very Short answer type questions carrying 2 marks each.
Answers to these questions should in the range of 30 to 50 words.
V) Section C consists of 7 short answer type questions carrying 3 marks each. Answer to
these questions should be in the range of 50 to 80 words.
VI) Section D Consist of 3 long answer type questions carrying 5 marks each. Answer to
these questions should be in the range of 80 to 120 words.
VII) Section E Consists of 3 source based/case study units of assessment of 4 marks each
with sub parts.

Section – A
Q.1) Shivani performed reaction in which the addition and removal of oxygen takes place
simultaneously. Choose the correct name for the reaction-

A. Oxidation reaction
B. Reduction reaction
C. Redox reaction
D. Precipitation reaction
Q. 2) Which of the following are exothermic processes?

i) Reaction of water with quick lime


ii) Dilution of an acid
iii) Evaporation of water
iv) Sublimation of Camphor

A. (i) and(ii)
B. (ii) and (iii)
C. (i) and (iv)
D. (iii) and ( iv)
Q.3) Vishal took 3ml of potassium sulphate solution in a beaker and added approximately 3ml of
barium chloride solution. What would be observed?

A. Insoluble Yellow precipitate


B. Insoluble green precipitate
C. Insoluble white precipitate
D. The soluble become blue.
Q. 4) The PH value about four substances A, B, C and D is a below-

A- 8.7
B- 5.5
C- 1.7
D- 12.3
Identify the basic compound from the above given substances.

A. (A) and ( B)
B. (B) and (C)
C. (A) and (D)
D. (C) and (D)

Q. 5) Chlor-alkali process is used for the manufacture of –

A. Chlorine
B. NaOH
C. Hydrogen
D. All of these
Q. 6) The ability of metals to be drawn into thin wires is known as-

A. Ductility
B. Malleability
C. Sonorousity
D. Conductivity
Q. 7) Identify the unsaturated compounds from the following-
I. Propane
II. Propene
III. Propyne
IV. Chloropropane
A. I and II
B. II and IV
C. III and IV
D. II and III
Q. 8) find out the incorrect statement given below :-

A. Length of small intestine in various animals depends upon the type of food they eat.
B. Enzymes for digestion of carbohydrates are not secreted in gastric juice.
C. In an Organism different enzymes have different pH.
D. Absorption of nutrients mostly occur in large intestine.
Q. 9) Trypsin Differ from pepsin in digestion of protein in-

A. Acidic medium
B. Alkaline medium
C. Neutral medium
D. All of the above
Q. 10) Which of the following acts as both endocrine and exocrine glands-

A. Adrenal
B. Pituatry
C. Ovaries
D. Pancreas
Q. 11) Which of the most common method of reproduction in majority of fungi and bacteria:-

A. Budding
B. Spore formation
C. Binary Fusion
D. Multiple fission
Q. 12) Segments of DNA which are responsible for pairs of an individual is called-

A. RNA
B. Gene
C. Chromosome
D. Any of the above
Q. 13) A torch bulb is rated at 1.5 V and 500 mA. It’s resistance will be-
A. 2 Ω
B. 2.5Ω
C. 1 Ω
D. 3 Ω
Q. 14) If a wire is stretched to make it’s length three time, it’s resistance will become –

A. Three time
B. One third
C. Nine times
D. D. One tenth.
Q. 15) The magnetic field due to a straight conductor carrying current-

A. Increases with the current flowing through it.


B. Decreases with the thickness of the conductor.
C. Increases as we move away from it.
D. Does not change with the change of material of the conductor.
Q. 16) The magnetic field due to a current carrying solenoid is :

A. Independent of the material of the coil.


B. Uniform inside the solenoid
C. Independent of the number of turns per unit Length of the solenoid.
D. Independent of the strength of flowing current.

Question No. 17 to 20
Assertion – reasoning based questions read the statement carefully and choose the correct
alternative from the following
A. both the assertion and reason are correct and the reason is the correct explanation of the
assertion
B. both assertion and reason are correct but reason is not the correct explanation of the
assertion
C. assertion is the true but the reason is false
D. assertion Is false but reason is true

17. Assertion – zinc reacts with sulphuric acid to from zinc sulphate and hydrogen gas and it is
displacement reaction.
Reason- zinc reacts with oxygen to form zinc oxide.
18. Assertion- Aerobic respiration require less energy as compared to anaerobic respiration
Reason – Mitochondria is the Power House of the cell

19. Assertion - When a compass needle is moved away from a current carrying straight
conductor the deflection of the needle decreases.
Reason - The strength of the magnetic field decreases on moving away from the straight
conductor.

20. Assertion - Acquired trait cannot be passed on from one generation to next generation
Reason - Inaccuracy during DNA copying of acquired trait is minimum.

Section-B
Q.21 Generally when metals are treated with mineral acids hydrogen gas is liberated but when
metals (except Mn and Mg) treated with HNO3 hydrogen is not liberated why.
OR
Name the following:-
1) A metal which can melt when kept on palm.
2) A metal alloyed with any another metal to make an amalgam

Q.22) How would digestion of food be affected if the bile duct is completely blocked explain?
Q.23) State the role of the following in human digestive system:-
I) Hydrochloric acid
II) Villi

Q. 24) State one example of each-

a) Positive phototropism
b) Chemotropism
Q.25 Gas A found in the upper layer of atmosphere is a deadly poison but is essential for all
living beings. The amount of this gas started decreasing sharply in the 1980s
A) Identify Gas A
B) Why is it essential for all living beings.
Q. 26) Name the part of human eye :-

a. That controls and regulates the amount of light entering the eye.
b. Has delicate membrane and contain large number of light sensitive cells.
OR
Give reasons :-

A) The extent of deviation of a ray of light on passing through a glass prism depends
on its colour.
B) Light of red colour is used for danger signals.

Section-C
Q. 27)

a) What changes in colour is observed When white silver chloride is left exposed to
sunlight.
b) State the type of chemical reaction involved in this change.
c) Write the chemical equation for this change.
Q. 28) State reasons for the following.

A) Dry hydrogen chloride gas does not turn blue litmus red whereas dilute
hydrochloric acid does.
B) During summer, a milk man usually adds a very small amount of baking soda to
fresh milk.
C) Ammonia is a base but does not contain hydroxyl group
Q. 29) Draw a diagram of human respiratory system and label trachea, lungs, diaphragm and
alveolar sac on it.
OR
The leaves of a plant were covered with Aluminum foil, how would it affect the physiology of
the plant?
Q. 30 A) What is understood by lateral displacement of light?
B) Illustrate it with the help of a diagram.
C) List any 2 factors on which lateral displacement of a particular substance depends?
Q.31) Ritu needs a lens of power -2D for correction of vision.

A) What kind of defect is she suffering from.


B) What are the possible causes of this defect.
C) What is the nature of corrective lens.
Q.32) State any 2 advantages and disadvantages of AC over DC.
OR
What is meant by solenoid? Draw magnetic field lines due to a current carrying conductor.
Q. 33) Explain biological magnification with the help of an example.

SECTION D
Q.34) Give reasons for the following:-

A) Carbon forms compounds mainly by covalent bonding.


B) Acetylene burns with a sooty flame.
C) Kerosene does not decolourise bromine water while cooking oils do.
D) Graphite is a good conductor of Electricity.
E) Soap form scum with hard water.
OR

Ethanoic acid reacts with absolute ethanol in the presence of Conc. H²SO⁴ to form a
compound.
A) Write the smell and class of compound to which this compound belong.
B) Write the chemical equation for the reaction.
C) State the role of conc. H²SO⁴ in the reaction.
D) Write one use of the product of this reaction.
Q. 35) Write the function of the following

A) Ovary
B) Uterus
C) Placenta
D) Seminal vesicles
E) Oviduct
OR

A) Name the two types of germ cells present in the human beings
B) How do they structurally differ from each other?
C) Why are testes located outside the abdominal cavity of the body.?
Q. 36) A hot plate of an electric oven connected to a 220 V has two resistance coils. A and b ,
each of 24 Ω resistance which may be use separately in series or in parallel .what are the current
in the three cases.

Section :- E

Q. 37) Atul and Riya were playing with a glass piece. They observe the image of a bird flying on
the wall of the room. The image formed on the wall was changing in size for the same bird as it’s
distance changes.

Q1) Which type of mirror is to be used to obtain the image of the bird on the wall?
Q2) Which type of lens can be used to obtain the image of the bird on the wall?
Q3) Two thin lenses of power 3.5 D and -2.5 D placed in contact. find the power and focal
length of the lenses in combination ?
OR
State the laws of refraction of light.

Q.38) The girls have not vanished overnight, decades of sex determination tests and female
foeticide raised the sex ratio in India.
All human beings have 23 pairs of chromosomes in the nucleus of their sales out of these only 2
chromosomes named X&Y are sex chromosomes.
These chromosomes determine the sex of the foetus. No body is responsible for such a natural
biological phenomenon.

Q1) What is the chromosomal difference between sperms and eggs of the human?
Q2) Why is sex determination banned?
Q3) How will the sex of the child be determined?
OR
What is meant by haploid and diploid?
Q. 39) The reactivity series offers a ranking of the metals in order of their reactivity. least
reactive metals are placed at the bottom while the highly reactive metals are at the top of the
series. More reactive metals can lose electrons more easily and form stable compound. More
reactive metals are difficult to extract from its ore.
Q1) Arrange the metals gold, copper, iron and magnesium in order of their increasing reactivity.
Q2) What will you observe when some silver pieces are put into green coloured ferrous sulphate
solution?
Q3) Common salt conduct electricity only in the molten state why?

OR
Why are Ionic compounds usually hard?
KENDRIYA VIDYALAYA SANGATHAN, JAMMU REGION
SAMPLE PAPER SET 02
CLASS - X SUBJECT : SCIENCE (086)
TIME : 3 hours M. M : 80

General instructions :
i. This question paper consists of 39 questions in 5 sections.
ii. All questions are compulsory. However, an internal choice is provided in some questions. A
student is expected to attempt only one of these questions.
iii. Section A consists of 20 objective type questions carrying 1 mark each.
iv. Section B consists of 6 Very Short questions carrying 02 marks each. Answers to these
questions should in the range of 30 to 50 words.
v. Section C consists of 7 Short Answer type questions carrying 03 marks each. Answers to
these questions should in the range of 50 to 80 words
vi. Section D consists of 3 Long Answer type questions carrying 05 marks each. Answer to
these questions should be in the range of 80 to 120 words.
vii. Section E consists of 3 source-based/case-based units of assessment of 04 marks each with
sub-parts.

Section A
Q1. A student took sodium sulphate solution in a test tube and added Barium Chloride solution
to it. He observed that an insoluble substance has formed. The colour and molecular formula of
the insoluble substance is:
(a) Grey, BaSO4
(b) Yellow, Ba(SO4)2
(c) White, BaSO4
(d) Pink, BaSO4
Q2. Which of the following are exothermic processes?
(i) Reaction of water with quick lime
(ii) Dilution of an acid
(iii) Evaporation of water
(iv) Sublimation of camphor (crystals)
(a) (i) and (ii)
(b) (ii) and (iii)
(c) (i) and (iv)
(d) (iii) and (iv)
Q3. Which one is a double displacement reaction?
(a) BaCl2 + Na2SO4——–> BaSO4 + 2NaCl
(b) CaO + H2O————> Ca(OH)2
(c) 2Pb(NO3)2 ——–> 2PbO + 4NO2 + O2
(d) Zn + 2AgNO3 ———-> Zn(NO3)2 + 2Ag
Q4. Which of the following salts do not have the water of crystalisation?
(i) Bleaching powder
(ii) sodium chloride
(iii) Washing soda
(iv) Baking soda
(a) (ii) and (iv). (b) (i) and (iii)
(c) (ii) and (iii). (d) (i) and (iv)
Q5. Which of the following gives the correct increasing order of acidic strength?
(a) Water <Acetic acid <Hydrochloric acid
(b) Water <Hydrochloric acid <Acetic acid
(c) Acetic acid <Water <Hydrochloric acid
(d) Hydrochloric acid <Water <Acetic acid
Q6. Which of the following metals is highly ductile:
(a) iron. (b) calcium
(c) Gold. (d) silver
Q7. Which of the following contains covalent bond?
(a) MgCl₂. (b) CaF2
(c) Al2 O3 (d) HCI
.
Q8. The opening and closing of the stomatal pore depends upon

(a)Oxygen
(b) Temperature
(c)Water at the guard cell
(d) Concentration of carbon dioxide.
Q9. The mode of nutrition found in fungi is:
(a) Parasitic nutrition
(b) Holozoic nutrition
(c) Autotrophic nutrition
(d) Saprotrophic nutrition
Q10. A student accidentally places her hand on a flame of candle and quickly pulls her hand
away. The flame represents
(a) a response (b) a stimulus
(c) an impulse. (d) an effector
Q11. Callus is defined as
(a) an insoluble carbohydrate b) a tissue which forms embryo
(c) a tissue which grows to form foetus (d) an unorganized mass of actively dividing cells
maintained in a culture.
Q12. If pea plants having round green seeds and wrinkled yellow seeds are crossed,what
phenotypic ratio will be obtained in F2 progeny plants.
(a) 1:2:1
(b) 3:1
(c) 9:3:3:1
(d) 9:3:3;5..

Q13. If R1 and R2 be the resistance of the filament of 40 W and 60 W, respectively operating


220 V, then
(a) R1 < R2
(b) R2 < R1
(c) R1 = R2
(d) R1 ≥ R2
Q14. In order to reduce electricity consumption at home, what kind of appliance should one
purchase?
(a) one which draws low power
(b) one which produces less heat
(c) one which operates at a higher voltage
(d) one which draws a high amount of current

Q15. Inside the magnet, the field lines moves

(a) from north to south

(b) from south to north

(c) away from south pole

(d) away from north pole

Q16 What is the relationship between resistance and current?

a)They are directly related to each other

b) They are inversely related to each other

c) The resistance has greater magnitude than current

d)The current has greater magnitude than resistance

For question numbers 17, 18 ,19 and 20, two statements are given-one labelled Assertion(A) and
other labelled Reason(R).Select the correct answer to these questions from the codes (a), (b), (c)
and (d) as given below.
A )Both A and R are true, and R is correct explanation of the assertion.
b) Both A and R are true, but R is not the correct explanation of the assertion
c)A is true, but R is false.
d) A is false, but R is true

Q 17)Asseretion:Mendal selected pea plant for his experiments


Reason:Pea plant is cross pollinating and has unisexual flowers
Q18. Assertion:Pyruvate is a six carbon molecule
Reason;It is prepared in the cytoplasm as the first step of cellular respiration.
Q19 Assertion:Calcium carbonate when heated gives gives calcium oxide and water.
Reason:On heating calcium carbonate ,decomposition reaction reaction takes place.

Q20. Assertion; When a charged particle enters in the direction of a uniform magnetic
field ,then it moves on a straight path without deviation.
Reason :Magnetic force on a charged particle is zero, when it moves in direction
. of magnetic field.

Section B- Short Answer Questions (02 marks each)


21. What are amphoteric oxides? Give two examples of amphoteric oxides.
Or
Define the following terms
a)Ductility
b)Malleability
22. What are the differences between the transport of materials in xylem and phloem?( Two
differences)
23. Write the function of the following in the human alimentary canal:
(a) Saliva. (b) bile juice
24. Answer the following:
(a) Which hormone is responsible for the changes noticed in females at puberty?-
(b) Blood sugar level rises due to deficiency of which hormone?-
(c) Dwarfism results due to deficiency of which hormone?
(d) Iodine is necessary for the synthesis of which hormone?
25. Draw a diagram of human eye and label the following parts: pupil, retina.
Or
Explain why a myopic person prefer to remove his spectacles while reading a book ?
26. State two problems caused by the non- biodegradable waste that we generate in our daily life.
Section C- Short Answer Questions (03 marks each)
27. Write the balanced chemical equations for the following reactions.
(a) Calcium hydroxide + Carbon dioxide → Calcium carbonate + Water
(b) Zinc + Silver nitrate. → Zinc nitrate + Silver
(c) Aluminium + Copper chloride → Aluminium chloride + Copper
28. (a) For the preparation of cakes, baking powder is used. If at home your mother uses baking
soda instead of baking powder, how will it affect the taste of the cake and why?
(b) How is baking soda be converted into baking powder?
(c) What makes the cake soft and spongy?
29. Explain the process of breakdown of glucose
(a) in the presence of oxygen. (b) in the absence of Oxygen

OR
What are 3 events in Photosynthesis ?.Explain
30. An object 4 cm in height, is placed at 15 cm in front of a concave mirror of focal length 10
cm. At what distance from the mirror should the screen be placed to obtain a sharp image of the
object. Calculate the height of the image.
31.a) Why does sky appear dark instead of blue to an astronaut?
b) Why do planets not twinkle?
32. Draw magnetic field lines around a bar magnet. List the properties of magnetic field lines.
Or
What happens to the force acting on current carrying conductor placed in a magnetic field when:
(a) Direction of magnetic field is reversed without changing the direction of current.
(b) Direction of the current is reversed without changing the direction of magnetic field.
(c) Direction of both the current and the magnetic field is reversed.
33. Gas A, found in the upper layers of the atmosphere, is a deadly poison but is essential for all
living beings. The amount of this gas started declining sharply in the 1980s.
(a) Identify Gas A. How is it formed at higher levels of the atmosphere?
(b) Why is it essential for all living beings? State the cause for the depletion of this gas.
Section D- Long Answer Questions (05 marks each)
34. Soaps and detergents are both types of salts. State the difference between the two. Write the
mechanism of the cleansing action of soaps. Why do soaps not form lather (foam) with hard
water? Mention any two problems that arise due to the use of detergents instead of soaps.
Or
(i) What change will you observe if you test soap with litmus paper (red & blue)?
(ii) What is Hydrogenation? What is its industrial application?
(iii)Give a test that can be used to differentiate between butter and cooking oil.

35. (a) Identify the given diagram. Name the parts from 1 to 5
(b) What is contraception? List three advantages of adopting contraceptive measures.
OR
a) How does embryo gets nourishment inside mother:s body
b) Where does fertilization take place in human females?
c)What is a seed and ovary ?
36. (a) 100 J of work is done in transferring 20 C of charge between two points in a conductor.
Find the resistance offered by the conductor, if a current of 2A flows through it
(b) Out of the two wires X and Y shown below, which one has greater resistance? Justify your

answer.

(c) What are the factors affecting resistance of a conductor?


Section E- Case Based Questions (04 marks each)
37. Read the given passage and answer the questions based on the passage and related studied
concepts.
Pure metals are usually too soft and weak for most uses. In pure metals the atoms are arranged
orderly in layers. When force is applies to the metal, the layers of metal atoms can slide over one
another.
To improve the strength and hardness of metals, atoms of another element can be added usually
in small amounts which prevents atoms of the metal from sliding over one another, making the
metals stronger and harder and less likely to get its shape distorted. The final product is an alloy
of metal, e.g. ornaments are made up of 22 carat gold in which copper is added to gold. Alloy is
a homogeneous mixture of two or more metals. One of them can be non-metal also, e.g., steel is
an alloy of Iron and carbon. Alloys are made so as to improve properties of metals. Amalgam is
alloy of metal with mercury.
(a) Stainless steel contains
(i) Fe + C (ii) Fe+C+ Cr
(iii) Fe+C+ Cr+ Ni. (iv) Fe+C+Mn+ Ni
(b) Solder is made up of
(i) Cu and Sn (ii) Pb and Sn
(iii) Zn and Cu (iv) Pb and Sb
(c) What are Alloys? How are Alloys made?
Or
a) What is the percentage of gold present in 22 carat gold ?
b) Name an alloy of Aluminium used for making body of aeroplanes
38. Answer the questions on the basis of your understanding of the following paragraph and
related studied concepts.
Within minutes of a baby's birth, people start remarking about who this new baby resembles. Oh,
he has his daddy's chin!' or 'She's got her mother's eyes!' but from where exactly do these
similarities arise? Every individual has 46 chromosomes, 23 chromosomes from each parent. The
chromosomes are composed of deoxyribonucleic acid or DNA that is tightly bundled. Now,
certain segments of the DNA which are responsible for different traits in an individual are
termed genes. Each chromosome contains over 20,000 genes. There is a lot of copying that has
to take place to pass all these genes on from parents to a newly growing child. Understandably
sometimes mistakes are made in the copying process.
(a) Number of chromosomes a human being has:
(i) 23 numbers
(ii) 23 pairs
(iii) 46 pairs
(iv) 34 numbers
(b) Which of the following is not true about variation?
(1) Variation forms the basis of heredity.
(ii) Variations enables organisms to adapt in changing environment.
(iii) Variation is more in case of asexual reproduction.
(iv) Variation leads to development of new species.
(c) How is sex of a child determined in human beings?
Or
c)What are genes? Where are they located?
39. Answer the questions that follow on the basis of your understanding of the following case
and the related studied concepts:
Shyam participated in a group discussion in his interschool competition on the practical
application of light and was very happy to won the award for his school. On that very evening,
his father celebrated the day with a family dinner. At a particular moment, Shyam observed in a
curve plate, the image of a person's mobile sitting on his back side. Person's mobile was fell off
which the person didn't know about it. Shyam went to the person and informed about this. The
person was thankful to Shyam.
(a) From which side of the plate Shyam observed the incident?
(i) Outward curved.
(ii) Inwards curved.
(iii) Plane surface.
(iv) The side of the plate which have both outward and inward curves alternately.
(b) The part of the curve plate from which Shyam observed the incidence acts like a
(i)concave mirror
(ii) convex mirror
(iii) plane mirror
(iv) concave-convex lens
(c) .Name the type of mirror used in
(i) Headlights of a car
(ii) Side/rear-view mirror of a vehicle
Or
(c) An object is placed at a large distance in front of a convex mirror of radius of curvature 40
cm. How far is the image behind the mirror?
KENDRIYA VIDYALAYA SANGATHAN, JAMMU REGION
SAMPLE PAPER SET 03
CLASS - X SUBJECT : SCIENCE (086)
TIME : 3 hours M. M : 80

General instructions :
i. This question paper consists of 39 questions in 5 sections.
ii. All questions are compulsory. However, an internal choice is provided in some questions. A
student is expected to attempt only one of these questions.
iii. Section A consists of 20 objective type questions carrying 1 mark each.
iv. Section B consists of 6 Very Short questions carrying 02 marks each. Answers to these
questions should in the range of 30 to 50 words.
v. Section C consists of 7 Short Answer type questions carrying 03 marks each. Answers to these
questions should in the range of 50 to 80 words
vi. Section D consists of 3 Long Answer type questions carrying 05 marks each. Answer to these
questions should be in the range of 80 to 120 words.
vii. Section E consists of 3 source-based/case-based units of assessment of 04 marks each with
sub-parts.
SECTION -A
Q1 The reaction in which two compounds exchange their ions to form two new compounds is
(a) a displacement reaction
(b) a decomposition reaction
(c) an isomerization reaction
(d) a double displacement reaction

Q2 Which reactant is reduced in given reaction


CuO + H2 → Cu + H2O
(a) Hydrogen
(b) Oxygen
(c) Copper Oxide
(d) none of the above

Q3 Magnesium ribbon is rubbed before burning because it has a coating of


(a) basic magnesium carbonate
(b) basic magnesium oxide
(c) basic magnesium sulphide
(d) basic magnesium chloride

Q4 Equal volumes of HCl and NaOH of same concentration are mixed and the PH of the resulting
solution is checked with PH paper. What would be the pH obtained

1
(A) 0
(B) 6
(C) 7
(D) 14

Q5. Common salt besides being used in kitchen can also be used as the raw material for
manufacturing
(i) washing soda (ii) bleaching powder (iii) baking soda (iv) slaked lime
(a) (i) and (ii)
(b) (i), (ii) and (iv)
(c) (i) and (iii)
(d) (i), (iii) and (iv)

Q6 Example of an amphoteric oxide is:


(a) Na2O
(b) K2O
(C) Al2O3
(d) MgO

Q7 Which of the following will give a pleasant smell of ester when heated with ethanol and a small
quantity of sulphuric acid?
(a) CH3COOH
(b) CH3CH2OH
(c) CH3OH
(d) CH3CHO

Q8 Identify the correct path of urine in the human body.


(a) Kidney → urinary bladder → urethra → ureter
(b) Urinary bladder → ureter → kidney → urethra
(c) Kidney → ureter → urethra → urinary bladder
(d) Kidney → ureter → urinary bladder → urethra

Q9 The mode of nutrition found in fungi is:


(a) Parasitic nutrition
(b) Holozoic nutrition
(c)Autotrophic nutrition
(d) Saprotrophic nutrition

Q10 What is missing?

(a) Recepter
(b) Muscle
(c) Spinal cord
(d) brain
2
Q11. The given diagram shows the carpel of an insect pollinated flower. What is the most likely
reason for the non-germination of pollen grain Z?

A) Pollen grains X and V were brought to the stigma earlier, therefore, their germination inhibited
the germination of pollen grain Z.

B) Pollen grain Z was brought to the flower by wind, while pollen grains X and Y were brought to
the flower by insects.

C) Pollen grain Z lacks protrusions that allow it to adhere properly onto the stigma surface.

D) Pollen grain Z comes from a flower of an incompatible species.


Q12 . In peas, a pure tall plant (TT) is crossed with a short plant (tt). The ratio of pure tall plants to
short plants in F2 is
a. 1: 3
b. 3: 1
c. 1: 1
d. 2: 1

Q13 Which of the following statements about the current in given circuit is correct?

A) Current at Q is greater than the current at R.


B) Current at R is greater than the current at P.
C) P has maximum current.
D) R has maximum current.

Q14 Two devices are connected between two points, say A and B in parallel. The physical quantity
that will remain the same between the two points is
a) current
b) voltage
c) resistance
d) None of these

3
Q15 . The figure given below shows the pattern of a magnetic field formed when current flows
through conductor.

Which of the following can form the pattern of the given magnetic field?

A) A solenoid
B) A straight wire
C) A coil
D) A bar magnet

Q16 . In India the potential difference between live wire and neutral wire is
(a) 240 V
(b) 250 V
(c) 280 V
(d) 220 V.

For question numbers 17, 18 ,19 and 20, two statements are given-one labelled Assertion(A) and
other labelled Reason(R).Select the correct answer to these questions from the codes (a), (b), (c)
and (d) as given below.
a) Both A and R are true, and R is correct explanation of the assertion.
b) Both A and R are true, but R is not the correct explanation of the assertion
c) A is true, but R is false.
d) A is false, but R is true
Q17 Assertion (A) : White silver chloride turns grey in sunlight.
Reason (R) : Decomposition of silver chloride in presence of sunlight takes place to form silver
metal and chlorine gas.

Q18 Assertion (A) : In anaerobic respiration, one of the end product is alcohol.
Reason (R) : There is an incomplete breakdown of glucose.

Q19. Assertion: Variations are seen in offspring produced by sexual reproduction.


Reason : DNA molecule generated by replication is not exactly identical to original DNA.

Q20 . Assertion: On changing the direction of flow of current through a straight conductor, the
direction of a magnetic field around the conductor is reversed.
Reason : The direction of magnetic field around a conductor can be given in accordance with left
hand thumb rule.
4
SECTION-B
Q21. (a) Why does calcium start floating when it reacts with water? Write the balanced chemical
equation of the reaction.
(b) Name two metals which do not react with water.
OR
a) Explain why Sodium and Potassium are stored in kerosene oil?
b)Explain why Ionic compounds have high melting point.?

22 I) Why is it necessary to separate oxygenated and deoxygenated blood in mammals and birds?
II) What is the role of saliva in the digestion of food?

Q23 (a) In the process of respiration, state the function of alveoli.


(b) Complete the following pathway showing the breakdown of glucose.

Q24. Why does the flow of signals in a synapse from axonal end of one neuron to dendritic end of
another neuron take place but not in the reverse direction?

Q25 Why do different components of white light deviate through different angles when passing
through a triangular glass prism?
OR
a)What is the far point and near point of the human eye with normal vision?
b)Why are danger signals red in colour?
Q26. Indicate the flow of energy in an ecosystem. Why is it unidirectional? Justify
SECTION-C
Q27 Mention with reason the colour changes observe when:
(i) silver chloride is exposed to sunlight.
(ii) copper powder is strongly heated in the presence of oxygen.

Q28 (a) A student dropped a few pieces of marble in dilute hydrochloric acid contained in a test
tube. The evolved gas was passed through lime water. What change would be observed in lime
water? Write balanced chemical equations for both the changes observed.
(b) State the chemical property in each case on which the following uses of baking soda are based:
5
(i) as an antacid
(ii) as a constituent of baking powder.

Q29 I) Leaves of a healthy potted plant were coated with Vaseline. Will this plant remain healthy
for long? Give reason for your answer.
ii) What is the significance of emulsification of fats?
OR

Give reason for the following:


(i) Why is diffusion not sufficient to meet the oxygen requirements of all the cells in multicellular
organisms?
(ii) How desert plants perform photosynthesis if their stomata remain closed during the day
Q30 A concave lens of focal length 15 cm forms an image 10 cm from the lens. How far is the
object placed from the lens? Draw the ray diagram.

Q31 (a) List two causes of hypermetropia.


(b) Draw ray diagrams showing (i) a hypermetropic eye and (ii) its correction using suitable optical
device.

Q32. i) Magnetic field lines are shown in the given diagram. A student makes a statement that
magnetic field at X is stronger than at Y. Justify this statement .

ii) Why two magnetic field lines never intersect each other
iii) Write the factors on which the strength of magnetic field lines depends in a current carrying
solenoid conductor
OR
i)What are magnetic lines of force?
ii)Write 2 differences between A.C and D.C
iii)What is the frequency of AC in India?

Q33 What is ozone? How and where is it formed in the atmosphere? Explain how it affects an
ecosystem?
SECTION-D
Q 34.i) Distinguish between esterification and saponification reactions with the help of chemical
equations for each. Write one use of each process
ii) Write the structural formula of ethanol. What happens when it is heated with excess of
concentrated Sulphuric acid at 443K.Write the chemical equation for reaction stating the role of
concentrated Sulphuric Acid?
6
OR
(a) Define the term catenation.
(b) What are the reasons for large number of carbon compounds?
(c) Why are carbon compounds poor conductors of electricity?

Q35. i) Suggest categories of contraceptive Methods to control the size of human population.
Also explain about each method briefly.
ii) Name two bacterial and two viral infections that can get sexually transmitted
OR
i. What is placenta? Mention its role during pregnancy
ii. Trace the path of male gamete takes to fertilise a female gamete after being released
from penis.
iii. Write number of sets of chromosomes present in a zygote

Q36. i) How can three resistors of resistances 2 Ω, 3 Ω, and 6 Ω be connected to give a total
resistance of (a) 4 Ω, (b) 1 Ω?
ii) An electric lamp of 100 Ω, a toaster of resistance 50 Ω, and a water filter of resistance 500 Ω are
connected in parallel to a 220 V source. What is the resistance of an electric iron connected to the
same source that takes as much current as all three appliances, and what is the current through it?
SECTION-E
Q37 Analyse the following reactivity series given below and answer the questions that follow

(a) Which of the two metals is more reactive: copper or silver?

(b) What will happen if a strip of zinc is immersed in a solution of copper sulphate?

(c) If copper is kept open in air, it slowly loses its shining brown surface and gains a green coating. Which
compound forms this green coating?
OR
(c) Which one of the metals (Mg, Ag, Zn and Cu) would be displaced from the solution of its salts by
other three metals?
Q38 . Mendel used a number of contrasting visible characteristics of garden peas- round or wrinkled
seeds, tall and short plants, white and Violet flowers and so on he took pea plants with different
7
characteristic : a tall plant and a short plant produced progeny from them and calculated the
percentage of tall or short progeny. Mendel chose pea plant for studying inheritance because pea
plants had a number of distinct differences which were easy to tell apart.
a) Give reason why only tall plants are observed in F1 progeny
OR
a)Why was Pea plant selected by Mendel? Give any two reasons

b)In monohybrid cross when F1 plants were self pollinated a total of 800 plants were produced.
How many of these would be tall plants and short plants. Give the Genotype of F2 generation
c)What is law of segregation.?
Q39 Light travel from one point to other along a straight path. This is called rectilinear propagation of
light. A bundle of rays constitutes a light beam. Reflection of light is a phenomenon of bouncing back
of light to the same medium after striking a surface. A glass sheet having a uniform thin coating of
silver on one side act as reflector and is called a mirror. Image of object formed by a plane mirror is
virtual and erect, same size as the object placed in front of it and laterally inverted. Refraction is the
phenomenon of bending of light ray incident obliquely at the interface of two different media

(a) An object is placed 20 cm in front of a plane mirror. The mirror is moved 2cm toward the
object. What is the distance between the position of the original and final images seen in
the mirror.
(b) What are the characteristics of image formed by plane mirror?
(c) A ray of light passes from a medium X to another medium Y. What should be the angle of
the ray of light hits the boundary of medium Y so, no refraction of light will occurs
OR
(c) What is the magnification produced by plane mirror?

8
KENDRIYA VIDYALAYA SANGATHAN, JAMMU REGION
SAMPLE PAPER SET 04
CLASS - X SUBJECT : SCIENCE (086)
TIME : 3 hours M. M : 80

General Instructions:
i. This question paper consists of 39 questions in 5 sections.
ii. All questions are compulsory. However, an internal choice is provided in some questions. A
student is expected to attempt only one of these choices.
iii. Section A consists of 20 objective type questions carrying 1 mark each.
iv. Section B consists of 6 Very Short questions carrying 02 marks each. Answers to these questions
should in the range of 30 to 50 words.
v. Section C consists of 7 Short Answer type questions carrying 03 marks each. Answers to these
questions should in the range of 50 to 80 words
vi. Section D consists of 3 Long Answer type questions carrying 05 marks each. Answer to these
questions should be in the range of 80 to 120 words.
vii. Section E consists of 3 source-based/case-based units of assessment of 04 marks each with sub-
parts.

Section A
1. Which of the following are exothermic processes?
(i) Reaction of water with quick lime
(ii) Dilution of an acid
(iii) Evaporation of water
(iv) Sublimation of camphor (crystals)
(a) (i) and (ii)
(b) (ii) and (iii)
(c) (i) and (iv)
(d) (ii) and (iv)
2. The process of reduction involves
(a) removal of hydrogen
(b) addition of oxygen
(c) addition of hydrogen
(d) None of these
3. What type of chemical reactions take place when electricity is passed through water?
(a) Displacement
(b) Combination
(c) Decomposition
(d) Double displacement
4. Precipitation reactions produce insoluble …………. .
(a) Sugar
(b) Salt
(c) Acid
(d) Base
5. Materials used in the manufacture of bleaching powder are
(a) lime stone and chlorine
(b) slaked lime and chlorine
(c) quick lime and chlorine
(d) slaked lime and HCl

Page 1 of 6
6. If pH of solution is 13, means that it is
(a) Strongly basic
(b) Weakly acidic
(c) Strongly acidic
(d) Weakly basic
7. Which among the following alloys contain non-metal as one of its constituents?
(a) Brass
(b) Amalgam
(c) Bronze
(d) Steel
8. Which of the following bonds is very strong and hence very stable?
(a) Carbon – Carbon bond
(b) Carbon – Iodine bond
(c) Carbon – Sulphur bond
(d) Carbon – Silicon bond

9. The image shows the bread moulds on a bread


How do these fungi obtain nutrition?

(a) by eating the bread on which it is growing


(b) by using nutrients from the bread to prepare their own food
(c) by breaking down the nutrients of bread and then absorbing them
(d) by allowing other organisms to grow on the bread and then consuming them

10. Main site of photosynthesis


(a) Leaf
(b) Stem
(c) Chloroplast
(d) Guard cells
11. The back flow of blood during the pumping of blood by heart is prevented by:
(a)Walls of ventncles
(b) Walls of atrium
(c) Valves in heart
(d) Walls between atria and ventricles
12. A person consuming sea food is least likely to develop
(a) Diabetese
(b) Goiter
(c) Both A and B
(d) Heart Diseases
13. Which part of the flower forms the fruit?

(a) Only ovary

Page 2 of 6
(b) Only carpel
(c) Only stamens
(d) Whole flower
14. A section of DNA providing information for one protein is called—
(a) Nucleus
(b) Chromosomes
(c) Trait
(d) Gene
15. Who have a perfect pair of sex chromosomes?
(a) Girls only
(b) Boys only
(c) Both girls and boys
(d) It depends on many other factors
16. The SI unit of electric potential difference is
(a) Volt
(b) Ampere
(c) Ohm
(d) Coulomb
Q. no 17 to 20 are Assertion – Reasoning based questions.

These consist of two statements – Assertion (A) and Reason (R). Answer these questions selecting the
Appropriate option given below:

(a) Both A and R are true and R is the correct explanation of A

(b) Both A and R are true and R is not the correct explanation of A

(c) A is true but R is false

(d) A is False but R is true


17. Assertion-The electric current flowing through the resistor is inversely proportional to it’s resistance.
Reasoning – Resistance is nothing to do with the flow of electric current.
18. Assertion - The human heart has 4 Chambers and only one chamber is involved in the pumping of
blood.
Reasoning – left ventricle pumps the blood to the entire body.
19. Assertion - The direction of induced current is given by Fleming’s Right Hand Rule.
Reasoning – induced current is the current produced by the forces of magnets.
20. Assertion – In a chemical reaction a new substance is not formed.
Reasoning – Combustion of wood is a chemical change.

Section B
21. Explain the meanings of malleable and ductile
22. Why do the walls of the trachea not collapse when there is less air in it?
23. What is the role of acid in our stomach?
OR
The sun can be seen about two minutes before actual sunrise and two minutes after actual sunset.
Why?
24. What is the function of receptors in our body? What happens when receptors do not work properly?
25. What is splitting of light? Explain it with the help of a diagram.
OR
How does rainbow formation takes place in atmosphere? Explain.
26. Write the common food chain of a pond ecosystem.

Page 3 of 6
Section C
27. Give one example each of :
(i) Thermal decomposition reaction
(ii) Electrolytic decomposition reaction
(iii) Photo decomposition reaction.
OR
What is the function of trachea ? Why do the walls not collapse even when there is less air in it ?
28. Why alkalies like sodium hydroxide and potassium hydroxide should not be left exposed to air?
29. Explain nutrition in amoeba.
OR
What are the types of blood vessels? Explain their functions.
30. Define the following terms in the context of spherical mirrors:
(i) Pole
(ii) Center of curvature
(iii) Principal axis
31. What is meant by scattering of light?
32. What are permanent magnet and electromagnets?
33. Distinguish between biodegradable and non-biodegradable wastes

Section D
34. What is methane? Draw its electron dot structure. Name the type of bonds formed in this compound.
Why are such compounds
(i) poor conductors of electricity and
(ii) have low melting and boiling points?
What happens when this compound burns in oxygen?
35. (a) Why is it not possible to reconstruct the whole organism from a fragment in complex
multicellular organisms?
(b) Sexual maturation of reproductive tissues and organs are necessary link for reproduction. Elucidate.
OR
(a) How are variations useful for species if there is drastic alteration in the niches?
(b) Explain how the uterus and placenta provide necessary conditions for proper growth and
development of the embryo after implantation?
36. (a) On what factors does the resistance of a conductor depends?
(b)Will current flow more easily through a thick wire or a thin wire of the same material, when
connected to the same source? Why?
OR
Page 4 of 6
(a) A current of 0.5 A is drawn by a filament of an electric bulb for 10 Minutes. Find the amount of
electric charge that flows through the Circuit.
(b) How much work is done in moving a charge of 2 C across two points having a potential
difference 12 V?
(c) Write the mathematical expression for finding the resistance in parallel.

Section E
37. Elements can be classified as metals or non-metals on the basis of their properties. The easiest way to
start grouping substances is by comparing their physical properties. Metals, in their pure state, have a
shining surface. This property is called metallic luster. Metals are generally hard. The hardness varies
from metal to metal. Some metals are used for making cooking vessels.
(i) The most abundant metal in the earth’s crust is –
(ii) The metal reacts with cold water is --
(iii) Metals present is chloroplast and haemoglobin are-
a) Iron and cobalt
b) Copper and zinc
c) Magnesium and iron
d) Cobalt and sodium
OR
(iv) Metals generally are:
a) Electropositive and reducing agent
b) Electropositive and oxidising agent
c) Both oxidising and reducing
d) None of these
38. The two sexes participating in sexual reproduction must be somewhat different from each other for a
number of reasons. How is the sex of a newborn individual determined? Different species use very
different strategies for this. Some rely entirely on environmental cues. Thus, in some animals like a few
reptiles, the temperature at which fertilised eggs are kept determines whether the animals developing in
the eggs will be male or female. In other animals, such as snails, individuals can change sex, indicating
that sex is not genetically determined. However, in human beings, the sex of the individual is largely
genetically determined. In other words, the genes inherited from our parents decide whether we will be
boys or girls.

(i) How many chromosomes are there in human beings?


(ii) What are the pair of sex chromosome present in both male and female?
(iii) How is the sex of the child determined in human beings?
OR
(iv) Give a cross between male and female for sex determination in human beings.
39. The lenses forms different types of images when object placed at different locations. When a ray is
incident parallel to the principal axis, then after refraction, it passes through the focus or appears to come
from the focus.
When a ray goes through the optical centre of the lens, it passes without any deviation. If the object
is placed between focus and optical center of the convex lens, erect and magnified image is formed.

Page 5 of 6
As the object is brought closer to the convex lens from infinity to focus, the image moves away from
the convex lens from focus to infinity. Also the size of image goes on increasing and the image is always
real and inverted.
A concave lens always gives a virtual, erect and diminished image irrespective to the position of
the object.

(i) The location of image formed by a convex lens when the object is placed at infinity is
(a) at focus
(b) at 2F
(c) at optical center
(d) between Fand 2F
(ii) When the object is placed at the focus of concave lens, the image formed is
(a) real and smaller
(b) virtual and inverted
(c) virtual and smaller
(d) real and inverted

(iii) What is the magnification and type of image formed in a plane mirror?
OR
(iv) When the object is placed at 2F in front of convex lens, what is the type and location of image?

Page 6 of 6
KENDRIYA VIDYALAYA SANGATHAN, JAMMU REGION
SAMPLE PAPER SET 05
CLASS - X SUBJECT : SCIENCE (086)
TIME : 3 hours M. M : 80

General Instructions:
i. This question paper consists of 39 questions in 5 sections.
ii. All questions are compulsory. However, an internal choice is provided in some questions. A student is expected to
attempt only one of these questions.
iii. Section A consists of 20 objective type questions carrying 1 mark each.
iv. Section B consists of 6 Very Short questions carrying 02 marks each. Answers to these questions should in the
range of 30 to 50 words.
v. Section C consists of 7 Short Answer type questions carrying 03 marks each. Answers to these questions should in
the range of 50 to 80 words
vi. Section D consists of 3 Long Answer type questions carrying 05 marks each. Answer to these questions should be
in the range of 80 to 120 words.
vii. Section E consists of 3 source-based/case-based units of assessment of 04 marks each with sub-parts.

SECTION -A

Q.1 When the gases sulphur dioxide and hydrogen sulphide mix in the presence of water, the reaction is SO2 +
2H2S → 2H2O + 2S. Here hydrogen sulphide is acting as –
(a) an oxidising agent
(b) a reducing agent
(c) a dehydrating agent
(d) a catalyst

Q.2 Which of the following are chemical changes?


(i) Digestion of food
(ii) Liquefaction of air
(iii) Ripening of fruit
(iv) Dissolution of sulphur in carbon disulphide
(v) Freezing of water
(vi) Electrolysis of water
(a) (i) to (iv) all
(b) (i), (iii) & (iv)
(c) (i), (iii) & (iv)
(d) (iii), (iv) & (iv)

Q.3 In which of the following, the identity of initial substance remains unchanged?
(a) Curdling of milk
(b) Formation of crystals by process of crystallisation
(c) Fermentation of grapes
(d) Digestion of food

Q.4 Which of the following can be decomposed by the action of light?


(a) NaCl
(b) KCl
(c) AgCl
(d) CuCl

Q.5 The self linking property of Carbon is called :


a) Catenation
b) Isomerisation
c) Bromination
d) None of these.

Q.6 A sample of soil is mixed with water and allowed to settle. The clear supernatant solution turns the pH paper
yellowish-orange. Which of the following would change the colour of this pH paper to greenish-blue:
(a) Lemon juice
(b) Vinegar
(c) Common salt
(d) An antacid

Q.7 The correct sequence of anaerobic reactions in yeast is


(a) Glucose Cytoplasm → Pyruvate mitochondria →Ethanol + Carbon dioxide
(b) Glucose Cytoplasm → Pyruvate Cytoplasm →Lactic acid
(c) Glucose Cytoplasm → Pyruvate mitochondria →Lactic acid
(d) Glucose Cytoplasm → Pyruvate Cytoplasm →Ethanol + Carbon dioxide

Q.8Which one of the following is a nastic movement in plants?


(a) Bending of plants towards light
(b) Growing of roots towards gravity.
(c) Dropping of touch – me – not leaflets on touch
(d) Movement of pollen tubes towards chemicals.

Q.9 Acetic acid was added to a solid X kept in a test tube. A colourless and odourless gas was evolved. The gas was
passed through lime water which turned milky. It was concluded that
(a) solid X is sodium hydroxide and the gas evolved is CO2
(b) solid X is sodium bicarbonate and the gas evolved is CO2
(c) solid X is sodium acetate and the gas evolved is CO2
(d) solid X is sodium chloride and the gas evolved is CO2

Q.10 The shorter fibres of nerve cell are called…


(a) dendrite
(b) axon
(c) synapse
(d) nerve fibre

Q.11 When an organism breaks into a number of parts and each part develop into an individual, it is called:
(a) Budding
(b) Binary fission
(c) Regeneration
(d) Spore formation

Q.12 Two pea plants one with round green seeds (RR yy) and another with wrinkled yellow (rrYY) seeds produce
F1 progeny that have round yellow (RrYy) seeds. When F1 plants are self pollinated, the F2 progeny will have a new
combination of characters. Choose the new combinations from the following:
(i) Round, yellow
(ii) Round, green
(iii) Wrinkled, Yellow
(iv) Wrinkled, green
(a) (i) and (ii)
(b) (i) and (iv)
(c) (ii) and (iii)
(d) (i) and (iii)

Q.13 Exchange of genetic material takes place in:


(a) Vegetative reproduction
(b) Asexual reproduction
(c) Sexual reproduction
(d) Budding

Q.14 The image shows a combination of 4 resistors.

What is the net resistance between the two points in the circuit?
(a) 0.5 Ω
(b) 1.0 Ω
(c) 1.5 Ω
(d) 2.0 Ω

Q.15 A TV set consumes an electric power of 230 watts and runs on 230 volts mains supply. The correct fuse for this
TV set is :
(a) 5 A
(b) 3 A
(c) 1 A
(d) 2 A

Q.16 A strong bar magnet is placed vertically above a horizontal wooden board. The magnetic lines of force will be
:
(a) only in horizontal plane around the magnet
(b) only in vertical plane around the magnet
(c) in horizontal as well as in vertical planes around the magnet
(d) in all the planes around the magnet
In the following questions a statement of assertion followed by a statement of reason is given. Choose the correct
answer out of the following choices.
a. Both Assertion and Reason are true and Reason is the correct explanation of Assertion.
b. Both Assertion and Reason are true but Reason is not the correct explanation of Assertion.
c. Assertion is true but Reason is false.
d. Both Assertion and Reason are false
Q.17 Assertion(A): On changing the direction of flow of current through a straight conductor, the direction of a
magnetic field around the conductor is reversed.
Reason (R) : The direction of magnetic field around a conductor can be given in accordance with left hand thumb
rule.

Q.18 Assertion (A) : Good conductors of heat are also good conductors of electricity and vice versa.
Reason (R) : Mainly electrons are responsible for conduction.

Q.19 Assertion: Presence of HCI in the stomach is necessary for the process of digestion.
Reason: HCI kills and inhibits the growth of bacteria in the stomach.

Q.20 Assertion: Sodium is kept immersed in kerosene oil.


Reason: Sodium is a very reactive metal.

SECTION -B

Q.21 Complete the following equation with balancing:


(i) Al + HCl →
(ii) Mg + HNO3 →
OR
Name the type of chemical reaction represented by the following equation:
(i) CaO + H2O → Ca(OH)2
(ii) 3BaCl2 + Al2(SO4)3 → 2AlCl3 + 3BaSO4

Q.22 State the role played by the following in the process of digestion :
(i) Enzyme trypsin
(ii) Enzyme lipase-

Q.23Diffusion is insufficient to meet the oxygen requirement of multicellular organisms like human. State reason.

Q.24 Define reflex arc.

Q.25 Explain the formation of Rainbow?


OR
Name the three common defects of vision. What is the cause of Myopia? Name the type of lens used to correct
Myopia.

Q.26 What are trophic levels? Give an example of food chain and state the different trophic levels in it.

SECTION -C

Q. 27 What do you mean by a precipitation reaction? Explain by giving examples.


Q.28 Write word equations and then balanced equations for the reaction taking place when
(a) dilute sulphuric acid reacts with zinc granules
(b) dilute hydrochloric acid reacts with magnesium ribbon
(c) dilute sulphuric acid reacts with aluminium powder

Q.29 Describe the structure and functions of nephrons.


OR
Reproduction is one of the most important characteristic ‘of living beings. Give three reasons in support of the
statement.

Q. 30 (a) State Right Hand Thumb rule to find the direction of the magnetic field around a current carrying straight
conductor.
(b) How will the magnetic field be affected on:
(i) increasing the current through the conductor
(ii) reversing the direction of flow of current in the conductor?

Q.31 What is meant by biological magnification?

Q.32 The image formed by a concave mirror is observed to be virtual, erect and larger than the object. Where should
the position of the object be relative to the mirror? Draw ray diagram to justify your answer.
OR
State the laws of refraction of light. Explain the term absolute refractive index of a medium’ and write an expression
to relate it with the speed of light in vaccum.

Q.33 The far point of a myopic person is 80 cm in front of the eye. What is the nature and power of the lens required
to correct the problem

SECTION -D

Q. 34 What is a homologous series? Explain with an example.


OR
Draw the structures for the following compounds:
(i) Ethanoic acid
(ii) Bromo pentane
(iii) Butanone
(iv) Hexanal
(v) cyclopentane

Q.35 Name the mode of reproduction of the following organisms and state the important feature of each mode :
(i) Planaria
(ii) Hydra
(iii) Rhizopus
(b) We can develop new plants from the leaves of Bryophyllum. Comment.
(c) List two advantages of vegetative propagation over other modes of reproduction.
OR

State the basic requirement for sexual reproduction. Write the importance of such reproductions in nature.
Q.36 What is meant by resistance of a conductor? Name and define its SI unit. List the factors on which the
resistance of a conductor depends. How is the resistance of a wire affected if – (i) its length is doubled, (ii) its radius
is doubled?

SECTION -E

CASE STUDY QUESTION

Question 37:
The spherical mirror forms different types of images when the object is placed at different locations. When the image
is formed on screen, the image is real and when the image does not form on screen, the image is virtual. When the
two reflected rays meet actually, the image is real and when they appear to meet, the image is virtual.
A concave mirror always forms a real and inverted image for different positions of the object. But if the object is
placed between the focus and pole. the image formed is virtual and erect.
A convex mirror always forms a virtual, erect and diminished image. A concave mirror is used as doctor’s head
mirror to focus light on body parts like eyes, ears, nose etc., to be examined because it can form erect and magnified
image of the object. The convex mirror is used as a rear view mirrors in automobiles because it can form an small
and erect image of an object.
(i) When an object is placed at the centre of curvature of a concave mirror, what will be nature of image

(ii) No matter how far you stand from a mirror, your image appears erect. The mirror is likely to be

(iii) A child is standing in front of a magic mirror. She finds the image of her head bigger, the middle portion of her
body of the same size and that of the legs smaller. The following is the order of combinations for the magic mirror
from the top.

OR

(iii) To get an image larger than the object which type of mirror we can use

Q. 38 Some metals are chemically very reactive, whereas others are less reactive or unreactive. On the basis of
vigourness of reactions of various metals with oxygen, water and acids, as well as displacement reactions, the metals
have been arranged in a group or series according to their chemical reactivity. The arrangement of metals in a
vertical column in the order of decreasing reactivities is called reactivity series of metals (or activity series of
metals). In reactivity series, the most reactive metal is placed at the top whereas the least reactive metal is placed at
the bottom. As we come down in the series, the chemical reactivity of metals decreases. Since the metals placed at
the bottom of the reactivity series (like silver and gold) are less reactive, so they are usually found in free
state (native state) in nature.

(i) Copper sulphate solution can be safely kept in a container made of _________________

(ii) When metal Z is added to dilute HCl solution, there is no evolution of gas. Metal is _____________

(iii) Metal always found in free state is :


a) Gold
b) Iron
c) Aluminium

OR
(iii) Which metal(s) catch fire on reaction with water?

Q.39 The rules for inheritance of such traits in human beings are related to the fact that both the father and the
mother contribute practically equal amounts of genetic material to the child. This means that each trait can be
influenced by both paternal and maternal DNA. Thus, for each trait there will be two versions in each child. What
will, then, the trait seen in the child be?

i) What were the contrasting traits used by mendel?


ii) What was the phenotypic ratio of monohybrid cross?
iii) What was the genotypic ratio of monohybrid cross?
OR
iii) How does the traits get expressed?
KENDRIYA VIDYALAYA SANGATHAN, JAMMU REGION
SAMPLE PAPER SET 06
CLASS - X SUBJECT : SCIENCE (086)
TIME : 3 hours M. M : 80

General Instructions:
i. This question paper consists of 39 questions in 5 sections.
ii. All questions are compulsory. However, an internal choice is provided in some
questions. A student is expected to attempt only one of these questions.
iii. Section A consists of 20 objective type questions carrying 1 mark each.
iv. Section B consists of 6 Very Short questions carrying 02 marks each. Answers to
these questions should in the range of 30 to 50 words.
v. Section C consists of 7 Short Answer type questions carrying 03 marks each.
Answers to these questions should in the range of 50 to 80 words
vi. Section D consists of 3 Long Answer type questions carrying 05 marks each. Answer
to these questions should be in the range of 80 to 120 words.
vii. Section E consists of 3 source-based/case-based units of assessment of 04 marks each
with sub-parts.
SECTION - A
Select and write one most appropriate option out of the four options given for each of
the questions 1 – 20
1. In the equation, NaOH + HNO3 -> NaNO3 + H2O nitric acid is acting as
(a)An oxidizing agent
(b)An acid
(c)A nitrating agent
(d)A dehydrating agent
2. Hydrogen sulphide (H2S) is a strong reducing agent. Which of the following reactions
shows its reducing action
(a)Cd(NO3)2 + H2S -> CdS + 2HNO3
(b)CuSO4 + H2S -> CuS + H2SO4
(c)2FeCl3 + H2S -> 2FeCl2 + 2HCI + S
(d)Pb(NO3)2 +H2S -> PbS + 2CH3COOH
3. In the following equation:
Na2CO3 + xHCI -> 2NaCI +CO2 + H2O, the value of x is
(a)1
(b)2
(c)3
(d)4
4. The reaction in which two compounds exchange their ions to form two new compounds
is
(a)A displacement reaction
(b)A decomposition reaction
(c)An isomerization reaction
(d)A double displacement reaction
5. The sample of soil from a particular place was tested for its pH value. It came out to be 5.
Which one of the following should be added to the soil to make it suitable for the plant
growth?
i. Calcium chloride
ii. Calcium Hydroxide
iii. Calcium oxide
Choose the correct option:
a. Both (i) and (ii)
b. Both (ii) and (iii)
c. Only (i)
d. Only (iii)
6. Copper sulphate crystals when heated strongly, lose their water of crystallization to give
anhydrous copper sulphate accompanied by a change in color from:
a. Blue to green
b. Blue to white
c. Blue to sky blue
d. Blue to grey
7. Which is the correct sequence of parts in human alimentary canal?
(a)Mouth →stomach →small intestine →oesophagus →large intestine
(b) Mouth →oesophagus →stomach →large intestine →small intestine
(c) Mouth →stomach →oesophagus →small intestine →large intestine
(d) Mouth →oesophagus →stomach →small intestine →large intestine
8. Rings of cartilage present in the throat ensure that
(a) air is filtered
(b) air is at room temperature
(c) air passage does not collapse
(d) air is free of microbes
9. Which of the following statements is not correct?
(a) Deoxygenated blood is poured into right atrium of heart.
(b) The excretory units of flatworms are flame cells.
(c) Human kidney has about 1 million nephridia.
(d) Tracheids and vessels are non-living conducting tissues.
10. A breeder crossed a pure-bred tall plant having white flowers to a pure bred short plant
having blue flowers. He obtained 202 F1 progeny and found that they are all tall having
white flowers. Upon selfing these F1 plants, he obtained a progeny of 2160 plants.
Approximately, how many of these are likely to be short and having blue flowers?
(a) 1215
(b) 405
(c) 540
(d) 135
11. In the experiment conducted by Mendel, RRyy (round green) and rrYY (wrinkled,
yellow) seeds of pea plant were used. In the F2 generation 240 progenies were produced, out
of which 15 progeny had specific characteristics. What were the characteristics?
(a) round and green
(b) round and yellow
(c) wrinkle and yellow
(d) wrinkle and green
12. The slope of voltage (V) versus current (I) is called

(a) resistance
(b) conductance
(c) resistivity
(d) conductivity
13. The element used almost exclusively for filaments of incandescent lamps
(a) copper
(b) gold
(c) silver
(d) tungsten
14. When a bar magnet is broken into two pieces?
(a) we will have a single pole on each piece
(b) each piece will have two like poles
(c) each piece will have two unlike poles
(d) each piece will be lose magnetism
15. Magnetic lines do not intersect on one-another because
(a) they are at a distance
(b) they are in the same direction
(c) they are parallel to another
(d) at the point of intersection there will be two direction of the magnetic force which is
impossible
16. The most suitable material for making the core of an electromagnet is:
(a) soft iron
(b) brass
(c) aluminium
(d) steel
Q. no 17 to 20 are Assertion - Reasoning based questions. These consist of two
statements – Assertion (A) and Reason (R). Answer these questions selecting the
appropriate option given below:
(a) Both A and R are true and R is the correct explanation of A
(b) Both A and R are true and R is not the correct explanation of A
(c) A is true but R is false
(d) A is False but R is true
17. Assertion (A) : Magnesium chloride is an ionic compound.
Reason (R) : Metals and non-metals react by mutual transfer of electrons.
18. Assertion(A): Cyton region of nerve fibre collects information for the brain.
Reason (R): Nerve fibres can either have or lack myelin sheath.
19. Assertion(A): Testes lie in penis outside the body.
Reason (R): Sperms require temperature lower than the body temperature for development
20. Assertion(A): Carbon compounds can form chain, branched and ring structures.
Reason (R): Carbon exhibits the property of catenation.

SECTION – B
Q. no. 21 to 26 are very short answer questions.
21. Name the following
(a) A metal which is preserved in kerosene
(b) A lustrous coloured non metal
OR
Why does a metal not release H2 gas on reacting with dilute nitric acid?
22. Ventricles have thicker muscular walls than atria. Give reason.
23. (i) Why are cramps caused in our muscles during sudden activity?
(ii) Name the type of respiration that takes place in Yeast during fermentation.
24. What are plant hormones? Name the plant hormones responsible for the following:
(i) Growth of stem
(ii) Promotion of cell division
25. Why do we see a rainbow in the sky only after rainfall?
OR
What is the colour of scattered sunlight when the size of the scattering particles is relatively
large?
26. The first trophic level in a food chain is always a green plant. Why?
SECTION - C
Q.no. 27 to 33 are short answer questions.
27. 2 g of ferrous sulphate crystals are heated in a boiling tube.
(i) State the colour of ferrous sulphate crystals both before heating and after heating.
(ii) Name the gases produced during heating.
(iii) Write the chemical equation for the reaction.
28. Two solutions A and B have pH 3 and 5 respectively. Which of the two solutions has
more hydrogen ion concentration and which one is more acidic? Give reason for your
answer.
29. Give reasons for the following:
(a) Arteries are thick walled.
(b) Blood goes only once through the heart in fishes.
(c) Plants have low energy needs.
OR
(a) State reason for the following:
(i) Rings of cartilage are present in the trachea.
(ii) Plants look green in colour.
(b) Write other names of the following:
(i) Alveolar sac
(ii) Voice box
30. An object is placed at the following distances from a concave mirror of focal length 15
cm.
(a) 10 cm
(b) 20 cm
(c) 30 cm
Which position of the object will produce
(i)Virtual image
(ii)A diminished real image
(iii)An enlarged real image
31. Define the term dispersion of white light. Name the colour of light which bends (i) the
most, (ii) the least, while passing through a glass prism. Draw a ray diagram to justify your
answer.
32. Explain the effect on the magnetic field produced at a point in a current carrying circular
coil due to:
(i) increase in the amount of current flowing through it
(ii) increase in the distance of point from the coil
(iii) increase in the number of turns of the coil.
OR
Give reasons for the following:
(a) It is dangerous to touch the live wire of the main supply rather than neutral wire.
(b) In household circuit, parallel combination of resistances is used.
(c) Using fuse in a household electric circuit is important.

33. What will be the amount of energy available to the organisms of the 2nd trophic level of a
food chain, if the energy available at the first trophic level is 10,000 Joules? Also explain the
reason.

SECTION - D
Q.no. 34 to 36 are Long answer questions.
34. Name the functional groups present in the following compounds:
(i) CH3—CH2—CH2—OH
(ii) CH3—CH2—CH2—COOH
(iii) CH3—CH2—CHO
(iv) CH3—CO—CH2—CH3
(v) CH3—CH2—CH3
OR
(a) What are homologous series of compounds? List any two characteristics of homologous
series.
(b) What would be observed by adding a 5% solution of alkaline potassium permanganate
drop by drop to warm ethanol taken in a test tube? Write the name of the compound formed
during the chemical reaction.
35. (a) Write the names of those parts of a flower which serve the same function as the
following do in the animals:
(i) testis (ii) sperm (iii) ovary (iv) egg
(b) State the function of flowers in the flowering plants.
OR
List five specific characteristics of sexual reproduction.
36. A bulb is rated at 200 V, 100 W. Calculate its resistance. Five such bulbs bum for 4 hours
daily. Calculate the units of electrical energy consumed per day. What would be the cost of
using these bulbs per day at the rate of ₹4.00 per unit?
SECTION - E
Q.no. 37 to 39 are case - based/data -based questions with 2 to 3 short sub - parts.
Internal choice is provided in one of these sub-parts.

37. Metallic money has been around for thousands of years, while paper money has only been
popular for a few hundred years. The first coins were worth their face value of whatever
precious metal they were made from. Today, all coins are deliberately made to be worthless
than their face value. All coins were originally made from gold, silver, and copper,
and these elements are still referred to as the coinage metals. Although some ancient coins
were sometimes made from pure metals, today, all coins intended for circulation are made
from alloys. The bronze alloy used to make coins today is typically composed of 95%copper,
4% tin, and 1% zinc.
The ubiquitous penny used to be made mostly of copper but is now mostly zinc. Zinc
is much less expensive than copper. Today’s penny is made up of 97.5% zinc, with a paper
thin copper coating that only makes up 2.5%of its total mass. There are several ways to
distinguish between old and new pennies. A post-1982 penny has a mass of 2.5 grams, while
the pre-1982 pennies have a mass of 3.1 grams.
Before 1982, if a small child swallowed a penny, doctors would generally advise to just
let it pass. With coins nowadays, little is as it appears. Not only are our “copper” pennies
mostly zinc, but our “silver” coins are mostly copper! Dimes and quarters minted before 1965
were composed of an alloy of 90% silver and10% copper, and they are considered somewhat
valuable by collectors. You can easily test for the presence of silver with a simple
experiment. Rub a little mustard (which naturally contains sulphur) on a silver coin and also
on a non-silver coin and let them stand overnight. In the morning, rub off the mustard.
(i) Why in this present era, the coins are deliberately made to be worth less than their face
value?
(a) due to poor market value of the metals used for making coins.
(b) to help and support the poor people of society
(c)to prevent them from being melted down and the metals recovered and sold.
(d) to have a check on black money.
(ii) What is the mass of the post-1982 penny?
(iii) Before 1982, if a small child swallowed a penny, doctors would generally advise to just
let it pass. Why?
OR
What is the chemistry behind the test for the presence of silver in a coin by rubbing with
mustard oil?
38.Rohan while visiting a zoo observed the fishes and crocodiles in different ponds. Sometimes
he observed crocodile seems to be smaller than its size inside water than when it comes out of
the water. On enquiring about the change in size of the crocodile he found the following
explanation for his query-
The apparent depth is the distance of the virtual image from the surface of the water. The real
depth is the distance of the real object from the surface of the water. The relation of refractive
index, n to real depth and apparent depth is given by: -
n = Real depth/ Apparent depth.
(i) A crocodile at the bottom of a pond appears to Rohan 1.1m from the water
surface. What is the depth of the pond? Refractive index of water=1.33.
(ii) Rohan was observing the coin placed at the bottom of a glass filled with water.
When he observed coin normally perpendicular to water surface, coin was almost
at its actual position instead of raised position. Explain why this happened?
OR
The figure below shows a boy standing inside a swimming pool. His legs appear
shorter when Rohan observes him from the pool side. Explain how this happen?

39. You may get to hear news about female foeticide or female infanticide. It is a cruel practice
to kill a girl child while it is still in the womb or just after her birth. Many people ostracise a
woman if she fails to bear a male child. People often blame the woman for this because of
ignorance. Many people put all the blame to the god. Let us try to understand how the gender
of a child is determined. In human beings, a somatic cell contains 23 pairs of chromosomes,
i.e., 46 chromosomes. Out of them, 22 pairs have similar chromosomes. These are called
autosomes. The chromosomes in the 23rd pair can be different and are called heterosome.
Chromosomes of the 23rd pair are also called sex chromosomes because they determine if a
zygote would develop into a male child or a girl child.
(i) Why is sex determination banned in India?
(ii) A child who inherits X chromosomes from father will be a
a) Boy
b) Girl
(iii) Why sex chromosomes are called heterosomes?
OR
What is meant by haploid and diploid?
KENDRIYA VIDYALAYA SANGATHAN, JAMMU REGION
SAMPLE PAPER SET 07
CLASS-X SUBJECT: SCIENCE
TIME : 3 hours M.M : 80

General Instructions
i. This question paper consists of 39 questions in 5 sections.
ii. All questions are compulsory. However, an internal choice is provided in some questions.
A student is expected to attempt only one of these questions.
iii. Section A consists of 20 objective type questions carrying 1 mark each.
iv. Section B consists of 6 Very Short questions carrying 02 marks each. Answers to these
questions should in the range of 30 to 50 words.
v. Section C consists of 7 Short Answer type questions carrying 03 marks each. Answers to
these questions should in the range of 50 to 80 words
vi. Section D consists of 3 Long Answer type questions carrying 05 marks each. Answer to
these questions should be in the range of 80 to 120 words.
vii. Section E consists of 3 source-based/case-based units of assessment of 04 marks each
with sub-parts.

1. What happens when a solution of an acid is mixed with a solution of a base in a test
tube?
(i) Temperature of the solution decreases
(ii) Temperature of the solution increases
(in) Temperature of the solution remains the same
(iv) Salt formation takes place
(a) (i) and (iv)
(b) (i) and (iii)
(c) (ii) only
(d) (ii) and (iv).

2.Which of the following are exothermic processes?


(i) Reaction of water with quick lime
(ii) Dilution of an acid
(iii) Evaporation of water
(iv) Sublimation of camphor (crystals)
(a) (i) and (ii)
(b) (ii) and (iii)
(c) (i) and (iv)
(d) (ii) and (iv)

3.When Ag is exposed to air it gets a black coating of


(a) AgNO3
(b) Ag2S
(c) Ag2O
(d) Ag2CO3
4.A dilute ferrous sulphate solution was gradually added to the beaker containing acidified
permanganate solution. The light purple colour of the solution fades and finally disappears.
Which of the following is the correct explanation for the observation?
(a) KMnO4 is an oxidising agent, it oxidises FeSO4.
(b) FeSO4 acts as an oxidising agent and oxidises KMNO4.
(c) The colour disappears due to dilution; no reaction is involved.
(d) KMnO4 is an unstable compound and de-composes in presence of FeSO4. to a colourless
compound.

5.Which of the following is the correct arrangement of the given metals in ascending order of
their reactivity?
Zinc, Iron, Magnesium, Sodium
(a) Zinc > Iron > Magnesium > Sodium
(b) Sodium > Magnesium > Iron > Zinc
(c) Sodium > Zinc > Magnesium > Iron
(d) Sodium > Magnesium > Zinc > Iron

6.An aqueous solution turns red litmus solution blue. Excess addition of which of the
following solution would reverse the change?
(a) Baking powder
(b) Lime
(c) Ammonium hydroxide solution
(d) Hydrochloric acid

7.Identify ‘A’ in the following reaction:


CH3COOH + Na2CO3 → A + CO2 + H2O
(a) CH3COONa
(b) CH2(Na)COOH
(c) NaOH
(d) NaHCO3

8. Which of the following events in the mouth cavity will be affected if salivary amylase is
lacking in the saliva?
(a) Starch breaking down into sugars.
(b) Proteins breaking down into amino acids.
(c) Absorption of vitamins.
(d) Fats breaking down into fatty acids and glycerol.

9.Which is the correct sequence of body parts in the human alimentary canal?
(a) Mouth → stomach → small intestine → large intestine → oesophagus
(b) Mouth → oesophagus → stomach → small intestine → large intestine
(c) Mouth → stomach → oesophagus → small intestine → large intestine
(d) Mouth → oesophagus → stomach → large intestine → small intestine

10.Two pea plants one with round green seeds (RR yy) and another with wrinkled yellow
(rrYY) seeds produce F1 progeny that have round yellow (RrYy) seeds. When F1 plants are
self pollinated, the F2 progeny will have a new combination of characters. Choose the new
combinations from the following:
(i) Round, yellow
(ii) Round, green
(iii) Wrinkled, Yellow
(iv) Wrinkled, green
(a) (i) and (ii)
(b) (i) and (iv)
(c) (ii) and (iii)
(d) (i) and (iii)

11. Identify which of the following statements about thyroxin is incorrect?


(a) Thyroid gland requires iodine to synthesize thyroxin.
(b) Thyroxin is also called thyroid hormone.
(c) It regulates protein, carbohydrates and fat metabolism in the body.
(d) Iron is essential for the synthesis of thyroxin.

12.Which of the following method of contraception protects from acquiring sexually


transmitted diseases?
(a) Surgery
(b) Condoms
(c) Copper-T
(d) Oral-pills

13.A circuit has a charge of 2C moving through it in 3 s. Which electrical component in the
circuit, if present, will show the current?
(a) Voltmeter will show a current of 6 A.
(b) Ammeter will show a current of 0.7 A.
(c) Rheostat will show a current of 0.7 A.
(d) Resistor will show a current of 0.35 A.

14.Which of the following statements is incorrect regarding magnetic field lines?


(a) The direction of the magnetic field at a point is taken to be the direction in which the
north pole of a magnetic compass needle points.
(b) Magnetic field lines are closed curves.
(c) If magnetic field lines are parallel and equidistant, they represent zero field strength.
(d) Relative strength of magnetic field is shown by the degree of closeness of the field lines.

15.A battery of 10 volt carries 20,000 C of charge through a resistance of 20 Ω. The work
done in 10 seconds is
(a) 2 × 103 joule
(b) 2 × 105joule
(c) 2 × 104 joule
(d) 2 × 102 joule

16.Which of the following correctly describes the magnetic field near a long straight wire?
a) The field consists of straight lines perpendicular to the wire
b) The field consists of straight lines parallel to the wire
c) The field consists of radial lines originating from the wire
d) The field consists of concentric circles centred on the wire
Q17 to 20 are Assertion - Reasoning based questions.
These consist of two statements – Assertion (A) and Reason (R). Answer these questions
selecting the appropriate option given below:
(a) Both A and R are true and R is the correct explanation of A
(b) Both A and R are true and R is not the correct explanation of A
(c) A is true but R is false
(d) A is False but R is true.

17.Assertion (A): Iron articles are painted so as to prevent them from rusting.
Reason (R): When the surface of iron is coated with paint, its surface does not come in
contact with oxygen and moisture therefore rusting does not take place.

18.Assertion : Accumulation of variation in a species increases the chances of its survival in


changing environment.
Reason : Accumulation of heat resistance in some bacteria ensure their survival even when
temperature in environment rises too much.

19.Assertion (A): The opening and closing of the pore is a function of the guard cells.
Reason (R) : Stomatal pores are the site for exchange of gases by diffusion.

20.Assertion(A): The magnitude of the magnetic field at a point on the axis of a current
carrying solenoid is inversely proportional to the current flowing through the solenoid.
Reason (R) : The magnitude of the magnetic field at a point on the axis of a current
carrying solenoid is directly proportional to the number of turns per unit length of a
solenoid.
Section B
21.What is meant by amphoteric oxides? Choose the amphoteric oxides from the following
Na2O, ZnO, CO2, Al2O3, H2O
Or
What happens when calcium oxide is dissolved in water? Write chemical equation of
reaction of the product so formed with carbon dioxide.
22. Why does the flow of signals in a synapse from axonal end of one neuron to dendritic
end of another neuron take place but not in the reverse direction? Explain.

23. Name the glands present in the wall of the stomach that release secretions for digestion
of food. Write the three components of secretion that are released by these glands.

24. State the role played by the following in the process of digestion :
(i) Enzyme trypsin
(ii) Enzyme lipase

25.What is the far point and near point of the human eye with normal vision?

Or
A person needs a lens of power -5.5 dioptres for correcting his distant vision. For correcting
his near vision he needs a lens of power +1.5 dioptre. What is the focal length of the lens
required for correcting (i) distant vision, and (ii) near vision?

26.Why do producers always occupy the first trophic level in every food chain?
Section C

27.1 g of copper powder was taken in a China dish and heated. What change takes place on
heating? When hydrogen gas is passed over this heated substance, a visible change is seen in
it. Give the chemical equations of reactions, the name and the colour of the products formed
in each case.

28. (a) Three acidic solutions A, B and C have pH = 1, 3 and 5 respectively.


i Which solution has highest concentration of H+ ions?
ii Which solution has the lowest concentration of H+ ions?
(b) How concentrated sulphuric acid can be diluted? Describe the process

29.Write a way in which glucose is oxidised to provide maximum energy in human body.
Write the two products formed in this case.

OR

Write three points of difference between respiration in plants and respiration in animals.

30. An object is situated at 8 cm from a convex lens of focal length 10 cm. Find the position
and nature of image. Draw ray diagram to illustrate the formation of image.

31. (i)Make a diagram to show that how you can use two identical prisms so that a narrow
beam of white light incident on one prism emerges out of second prism as white light?
(ii)Why do we see a rainbow in the sky only after rainfall?

32.Define a solenoid. Compare the magnetic field produced by a solenoid with that of a bar
magnet?
OR

What are magnetic field lines? How is the direction of a magnetic field at a point determined?
Mention two important properties of the magnetic field lines.

33.(a) From the following group of organisms create a food chain which is most
advantageous for human beings in terms of energy.

Hawk, Rat, Cereal plant, Goat, Snake, Human being


(b) State the possible disadvantage if the cereal plant is growing in soil rich in pesticides.
(c) Construct a food web using the organisms mentioned above.

Section D.

34.i) Write one chemical equation to represent each of the following types of reactions of
organic substances.
(a) Esterification
(b) Saponification
(c) Substitution
ii ) Distinguish between ethanol and ethanoic acid on the basis of (i) litmus test, (ii) reaction
with sodium hydrogen carbonate
OR
A carboxylic acid C2H4O2 reacts with an alcohol in the presence of H2SO4 to form a
compound ‘X’. The alcohol on oxidation with alkaline KMnO4 followed by acidification
gives the same carboxylic acid, C2H4O2. Write the name and structure of: (i) Carboxylic acid,
(ii) alcohol and (iii) the compound ‘X’.

35.i) What happens when


(a) Planaria gets cut into two pieces
(b) a mature Spirogyra filament attains considerable length
(c) on maturation sporangia burst?
ii)What are the functions of testes in the human male reproductive system? Why are these
located outside the abdominal cavity? Who is responsible for bringing about changes in
appearance seen in boys at the time of puberty?

OR
(i)State the changes that take place in the uterus when:
(a) Implantation of embryo has occurred.
(b) Female gamete/egg is not fertilised.

(ii)List three techniques that have been developed to prevent pregnancy. Which one of these
techniques is not meant for males? How does the use of these techniques have a direct impact
on the health and prosperity of a family?

36a).i :List three factors on which the resistance of a conductor depends.


(ii) Write the SI unit of resistivity.
b)Calculate the resistance of a metal wire of length 2m and area of cross section 1.55 × 106
m², if the resistivity of the metal be 2.8 × 10-8 Ωm.

Section E
37:Rohit went to a restaurant with his mother and ordered so many things to eat. While his
orders, the waiter offered them with water. He started drinking water using a straw, and
suddenly he kept on looking inside the glass in a very fascinating way. His mother asked him,
’’What are you looking inside. Then he replied that straw is looking bent. Mother explained
him that it’s a scientific concept explained by refraction.
37 a)What is the cause of Refraction ?
37 b) What happens to the speed of light when it goes from denser to the rarer medium
37 c) Light enters from air to glass having refractive index 1.50. what is the speed of light in
glass? The speed of light in vacuum is 3X 108m/s.

OR
What is the relation of refractive index(n) of medium and speed of light(v) ?

38.India is a country where people are very fond of wearing various ornaments specially
made up of metals like gold, silver and platinum.In Mohan family there was marriage of his
sister. So one day he and his parents went to a goldsmith to buy some jewellery for his sister.
He saw a worker there beating gold into sheet and wires to make beautiful jewellery. He
discussed this observation with his teacher. His teacher told him about the fact behind it.It is
property of metals to be beaten into sheets called malleability and to be stretched into wires
called ductility. These are physical properties of metal. Mohan was now aware about that fact
seen in goldsmith shop.

38(a) Aluminium foil is sheet of metal which is used to wrap food item. Which property of
Metals is used as foil? the following property is being used here.
(b) Which is the most ductile metal?
(c) What is that only solution with which gold can react name it and write its constituents
with ratio
OR

Write a metal and a non metal in liquid state?

39. Pea plants can have smooth seeds or wrinkled seeds. One of the phenotypes is completely
dominant over the other. A farmer decides to pollinate one flower of a plant with smooth seeds
using pollen from plant with wrinkled seeds just by following emasculation. The resulting pea
pod has all smooth seeds. So, On the basis of his experiments answer the following:

39(a) What name is given to the phenotypic characters of F1 generation?

39(b) On crossing of two heterozygous smooth seeded plants (Rr), If a total of 1000 plants
were obtained in F1 generation What will be the respective number of smooth and wrinkled
seeds obtained in F1 generation

39(c) Out of the given two crosses what do you think which will give smooth and wrinkled

Seeds in same proportion (1) RR X rr or (2) Rr X rr. Justify.

OR

How can you incorporate Mendel’s experiment here that traits are inherited

independently. Support your answer with example.


KENDRIYA VIDYALAYA SANGATHAN, JAMMU REGION
SAMPLE PAPER SET 08
CLASS - X SUBJECT : SCIENCE (086)
TIME : 3 hours M. M : 80
General Instructions:
i. This question paper consists of 39 questions in 5 sections.
ii. All questions are compulsory. However, an internal choice is provided in some questions. A
student is expected to attempt only one of these questions.
iii. Section A consists of 20 objective type questions carrying 1 mark each.
iv. Section B consists of 6 Very Short questions carrying 02 marks each. Answers to these
questions should in the range of 30 to 50 words.
v. Section C consists of 7 Short Answer type questions carrying 03 marks each. Answers to these
questions should in the range of 50 to 80 words
vi. Section D consists of 3 Long Answer type questions carrying 05 marks each. Answer to these
questions should be in the range of 80 to 120 words.
vii. Section E consists of 3 source-based/case-based units of assessment of 04 marks each with
sub-parts.
SECTION A
Select and write one most appropriate option out of the four options given for each of the
questions 1 – 20
Q1 A circuit has a charge of 2C moving through it in 3 s. Which electrical component in the
circuit, if present, will show the current?
(a) Voltmeter will show a current of 6 A.
(b) Ammeter will show a current of 0.7 A.
(c) Rheostat will show a current of 0.7 A.
(d) Resistor will show a current of 0.35 A
Q2 A researcher adds barium hydroxide to hydrochloric acid to form a white-coloured barium
chloride. Which option gives the balanced chemical equation of the reaction?
(a) HCl + Ba(OH)2 → BaCl2 + 2HOH
(b) 2HCl + Ba(OH)2 → BaCl2 + 2HOH
(c) 2HCl + Ba(OH)2 → BaH2 + 2HCl + O2
(d) HCl + 2Ba(OH) → 2BaCl2 + 2HOH + O2
Q3 When a magnesium ribbon is burnt in air, the formed is
(a) black
(b) white
(c) yellow
(d) pink
Q4 . An element X on exposure to moist air turns reddish-brown and a new compound Y is formed. The
substance X and Y is
(a) X = Fe, Y = Fe2O3
(b) X = Ag, Y = Ag2S
(c) X = Cu, Y = CuO
(d) X = Al, Y = Al2O3
Q5 Na2CO3.10H2O is known as -
(a) Baking Soda
(b) Baking Powder
(c) Washing Soda
(d) Bleaching Powder
Q6 What is the chemical formula of POP (Plaster of Paris)?
(a) CaSO4.2H2O
(b) CaSO4.3H2O
(c) CaSO4.1/2H2O

(d) CaCO3.1/2H2

Q7 Aluminium is used for making cooking utensils. Which of the following properties of
aluminium are responsible for the same? 1
(i) Good thermal conductivity
(ii) Good electrical conductivity
(iii) Ductility
(iv) High melting point
a) (i) and (ii)
b) (i) and (iii)
c) (ii) and (iii)
d) (i) and (iv)
Q8 The image represents the structure of a few hydrocarbon compounds. 1

Which of these compounds can be classified as alkynes?


(a) only (A)
(b) only (B)
(c) both (A) and (D)
(d) both (B) and (C)
Q9 How will information travel within a neuron? 1
(a) Dendrite -> cell body -> axon -> nerve ending
(b) Dendrite -> axon -> cell body -> nerve ending
(c) Axon -> dendrite -> cell body -> nerve ending
(d) Axon -> cell body -> dendrite -> nerve ending
Q10 Which of the following correctly describes the magnetic field near a long straight wire?
a) The field consists of straight lines perpendicular to the wire
b) The field consists of straight lines parallel to the wire
c) The field consists of radial lines originating from the wire
d) The field consists of concentric circles centred on the wire
Q11 Which region of the alimentary canal absorbs the digested food?
(a) Stomach
(b) Small intestine
(c) Large intestine
(d) Liver
Q12 What are the products obtained by anaerobic respiration ?
(a) Lactic acid + Energy
(b) Carbon dioxide + Water + Energy
(c) Ethanol + Carbon dioxide + Energy
(d) Pyruvate
Q13 In the below figure the parts A, B and C are sequentially

a) Cotyledon, plumule and radicle


b) Plumule, radicle and cotyledon
c) Plumule, cotyledon and radicle
d) Radicle, cotyledon and plumule
Q14The factors on which one magnetic field strength produced by current carrying solenoids depends
are
(a) Magnitude of current
(b) Number of turns
(c) Nature of core material
(d) All of the above
Q15 Which of the following diseases is transmitted sexually?
a) Kala Azar
b) Jaundice
c) Cholera
d) Syphillis
Q16 Calculate the current flows through the 10 Ω resistor in the following circuit.

(a) 1.2 A
(b) 0.6 A
(c) 0.2 A
(d) 2.0 A

Q. no 17 to 20 are Assertion - Reasoning based questions.

These consist of two statements – Assertion (A) and Reason (R). Answer these questions selecting the

appropriate option given below:

(a) Both A and R are true and R is the correct explanation of A

(b) Both A and R are true and R is not the correct explanation of A

(c) A is true but R is false

(d) A is False but R is true


Q17 . Assertion (A) – White silver chloride turns grey in sunlight.
Reason (R) – Decomposition of silver chloride in presence of sunlight takes place to
form silver metal and chlorine gas.
Q18 Assertion(A) : Variations are seen in offspring produced by sexual reproduction.
Reason (R) : DNA molecule generated by replication is not exactly identical to original DNA.

Q19 Assertion (A) : Carbohydrate digestion mainly takes place in small intestine.
Reason (R) : Pancreatic juice contains the enzyme lactase.
Q20 Assertion(A) : A compass needle is placed near a current carrying wire. The deflection
of the compass needle decreases when the compass needle is displaced away from the wire.
Reason (R) : Strength of a magnetic field decreases as one moves away from a current
carrying conductor.
SECTION – B
Q21-Q26 are very short answer questions
21. Complete the following equation with balancing:
(i) Al + HCl →
(ii) Mg + HNO3 →
OR
Why does calcium start floating when it reacts with water? Write the balanced chemical equation of
the reaction.

Q22 What is the role of saliva in the digestion of food ?


Q23What is excretion ? How do unicellular organisms remove their wastes ?
Q24 What will happen if intake of iodine in our diet is low?
Q25 What is short sightedness? How can it be corrected?
OR
Why do we see a rainbow in the sky only after rainfall?

Q26 List two advantages of decomposers to the environment.


SECTION – C
Q27 - Q33 are short answer questions
Q27 Write balanced chemical equations for the following chemical reactions:
(a) Hydrogen + Chlorine → Hydrogen chloride
(b) Lead + Copper chloride → Lead chloride + Copper
(c) Zinc oxide + Carbon → Zinc + Carbon monoxide
Q28 What are strong and weak acids? In the following list of acids, separate strong acids from weak acids.
Hydrochloric acid, citric acid, acetic acid, nitric acid, formic acid, sulphuric acid.
Q29 What are the functions of the liver and the pancreas?
OR
Draw a well labelled diagram of human excretory system?

Q30 A concave lens of focal length 15 cm forms an image 10 cm from the lens object placed from the lens?
Draw the ray diagram.
Q31 Explain clearly the working of a human eye?
Q32 Give important advantage and disadvantages of AC over DC.
OR
What is solenoid? Draw the pattern of magnetic field lines of a current carrying solenoid

Q33 What are decomposers? List two important roles they play in the environment.
SECTION – D
Q34 - Q36 are long answer question
Q34 Give reasons for the following:
(i) Element carbon forms compounds mainly by covalent bonding.
(ii) Diamond has high melting point.
(iii) Graphite is a good conductor of electricity
Q35(a) Name the mode of reproduction of the following organisms and state the important feature of each mode

(i) Planaria
(ii) Hydra
(iii) Rhizopus
(b) We can develop new plants from the leaves of Bryophyllum. Comment.
(c) List two advantages of vegetative propagation over other modes of reproduction.
OR
What are the functions of testes in the human male reproductive system? Why are these located outside
the abdominal cavity? What is responsible for bringing about changes in appearance seen in boys at the
time of puberty?
Q36 What is meant by resistance of a conductor? Name and define its SI unit. List the factors on which
the resistance of a conductor depends. How is the resistance of a wire affected if: 5
(i) its length is doubled
(ii) its radius is doubled?

OR
State ohms law. Represent it graphically. In the given circuit diagram calculate
(i) the total effective resistance of the circuit.
(ii) the current through each resistor.

SECTION – E
Q37 - Q39 are case based / data based question
Q37 Ores mined from the earth are usually contaminated with large amounts of impurities such as soil, sand,
etc., called gangue. The impurities must be removed from the ore prior to the extraction of the metal. The
processes Several steps are involved in the extraction of pure metal from ores. Metals and Non-metals used for
removing the gangue from the ore are based on the differences between the physical or chemical properties of
the gangue and the ore. Different separation techniques are accordingly employed. (1+2+1)
i)What are the process for the extraction of metals of low reactivity series?
ii)What is difference between Roasting and Calcination?
OR
Why are only few metals used to make jewellery? What is the name given to the blackening of silver on
exposure to the atmosphere
iii) Give one example of extracting metals low in the activity series.

Q38 Read the following and answer any four questions from (i) to (v).
Pea plants can have smooth seeds or wrinkled seeds. One of the phenotypes is completely dominant
over the other. A farmer decides to pollinate one flower of a plant with smooth seeds using pollen
from plant with wrinkled seeds. The resulting pea pod has all smooth seeds (1++1+1+1)

38.1 What is the name given to trait in first filial generation

38.2 Out of the given two crosses what do you think which will give smooth and wrinkled Seeds in
same proportion (1) RR X rr or (2) Rr X rr. Justify.

OR

How can you incorporate Mendel’s experiment here that traits are inherited
38.3On crossing of two heterozygous smooth seeded plants (Rr), If a total of 1000 plants were
obtained in F1 generation What will be the respective number of smooth and wrinkled seeds
obtained in F1 generation
Q39 Analyse the following table showing refractive indexes of four media A,B,C and D are
given in the following table.
Medium A B C D
Refractive
1.33 1.50 1.52 2.4
Index

i) If light travels from one medium to another medium given in the above table, in which case the
change in the speed will be minimum?
ii) If light travels from one medium to another medium given in the above table, in which case
the change in the speed will be maximum?
iii) A light ray enters from medium A to medium B as shown in the figure below. The refractive
index of medium B with relative to A will be:

(a) Greater than unity


(b) Less than unity
(c) Equal to unity
(d) Zero
OR

(A) The laws of reflection hold good for___________________ type/types of mirrors.


KENDRIYA VIDYALAYA SANGATHAN, JAMMU REGION
SAMPLE PAPER SET 09
CLASS - X SUBJECT : SCIENCE (086)
TIME : 3 hours M. M : 80
General instructions:
i. This question paper consists of 39 questions in 5 sections.
ii. All questions are compulsory. However, an internal choice is provided in some
questions. A student is expected to attempt only one of these questions.
iii. Section A consists of 20 objective type questions carrying 1 mark each.
iv. Section B consists of 6 Very Short questions carrying 02 marks each. Answers to
these questions should in the range of 30 to 50 words.
v. Section C consists of 7 Short Answer type questions carrying 03 marks each.
Answers to these questions should in the range of 50 to 80 words.
vi. Section D consists of 3 Long Answer type questions carrying 05 marks each.
Answer to these questions should be in the range of 80 to 120 words.
vii. Section E consists of 3 source-based/case-based units of assessment of 04 marks
each with sub-parts.

S.NO. QUESTIONS.
SECTION A. Marks.
Section- A consists of 20 question carrying 1 marks each.( 1×20= 20).

1. A magnesium ribbon is burnt in air. A student made the following observations: 1 mark
i. Magnesium ribbon undergoes oxidation reaction.
ii. Magnesium reacts with air to form magnesium oxide.
iii. Magnesium ribbon undergoes decomposition reaction.

Which of the following statements is correct?

(a)(ii) and (iii) (b) (i) and (iii)


(c)(i), (ii) and (iii) (d) (i) and (ii)

2. Physical changes are accompanied by: 1 mark

(a) Evolution of hydrogen gas


(b) Change in state
(c) Formation of a new yellow precipitate
(d) Formation of a new compound.
3. The figure given below represents 1 mark
the experiment carried out between
conc. sulphuric acid and sodium
chloride, which react with each
other to form HCl gas. Blue litmus
paper is brought near the mouth of
the delivery tube to check the
presence of HCl acid but no change
is observed in the colour of litmus
paper because:

(a) The litmus paper used is dry.


(b) The litmus paper used is moist.
(c) Blue litmus paper does not change its colour with an acid.
(d) The litmus paper is kept very close to the mouth of the delivery tube.

4. A substance ‘X’ is used in white-washing and is obtained by heating limestone in the 1 mark
absence of air. Identify ‘X’.
(a) CaOCl2
(b) Ca (OH)2
(c) CaO
(d) CaCO3

5. The graph given below depicts a neutralization reaction (acid + alkali → salt + water). 1 mark
The pH of a solution changes as we add excess of acid to an alkali.

Which letter denotes the area of the graph where both acid and salt are present?

(a) A
(b) B
(c) C
(d) D
6. Metals_______electrons while Non metals_____ electrons. 1 mark

(a) Accept, Donate.


(b) Donate, Accept.
(c) Did not have, have.
(d) None of these.

7. Which of the following statements is not true for stomatal apparatus? 1 mark

(a) Guard cells invariably possess chloroplasts and mitochondria.


(b) Guard cells are always surrounded by subsidiary cells.
(c) Stomata are involved in gaseous exchange.
(d) Inner wall of guard cells are thick.

8. The given flow chart shows the fate of carbohydrates during digestion in the human 1 mark
alimentary canal. Identify the enzymes acting at stages indicated as A, B, C and D and
select the correct option.

(a) A-Amylase, B- Maltase, C-Lactase, D - Invertase


(b) A-Amylase, B-Maltase, C- Invertase, D- Lactase
(c) A-Amylase, B-Invertase, C-Maltase, D - Lactase
(d) A-Amylase, B-Lactase, C-Maltase, D - Invertase

9. In a synapse, chemical signal is transmitted from 1 mark


(a) Dendritic end of one neuron to axonal end of another neuron
(b) Axon to cell body of the same neuron
(c) Cell body to axonal end of the same neuron
(d) Axonal end of one neuron to dendritic end of another neuron.

10. In Rhizopus, tiny blob-on-a-stick structures are involved in reproduction are called 1 mark
(a) Filament
(b) Hyphae
(c) Rhizoids
(d) Sporangia
11. A man with blood group B marries a women with blood group A and their first child is 1 mark
having blood group B. What is the genotype of child_____.
(a) I A I B
(b) I A I O
(c) I B I O
(d) I B I B

12. The equivalent resistance in this circuit is____? 1 mark

(a) 100 ohms


(b) 450 ohms
(c) 400 ohms
(d) 30 ohms

13. The front face of a circular loop of a wire is North-pole, the direction of current in this
face of the loop will be:
(a) Clockwise
(b) Anticlockwise
(c) Towards North
(d) Towards South

14. If you break a bar magnet in half, each half.

(a) Contains one magnetic pole.


(b) Becomes a bar magnet with two poles.
(c) Becomes unmagnetized.
(d) Is only temporarily magnetized.

15. Magnetic field inside a long solenoid carrying current is____?


(a) Same at all points (uniform)
(b) Different at poles and at the centre
(c) Zero
(d) Different at all points

16. A wire of length l, made of material resistivity ρ is cut into two equal parts. The
resistivity of the two parts are equal to,
(a) ρ
(b) ρ2
(c) 2 ρ
(d) 4 ρ

Question No. 17 to 20 consist of two statements – Assertion (A) and Reason (R). 1 mark
Answer these questions selecting the appropriate option given below:
A. Both A and R are true and R is the correct explanation of A
B. Both A and R are true and R is not the correct explanation of A
C. A is true but R is false
D. A is False but R is true

17 Assertion: The process of dissolving an acid or a base in water is highly exothermic 1 mark
reaction.
Reason: Water must always be added slowly to acid with constant stirring.

18. Assertion : The movement of water and dissolved salts in xylem is always upwards. 1 mark
Reason: 'The upward movement of water is due to negative pressure created by
transpiration.

19. Assertion: The functional group present in alcohols is – OH. 1 mark


Reason: It is the same group as present in water, hence water and alcohol have
similar properties.

20. Assertion: Acquired trait cannot be passed on from one generation to next generation. 1 mark
Reason: Inaccuracy during DNA copying of acquired trait is minimum.

SECTION – B
Section- B consists of 6 question carrying 2 marks each.( 2×6= 12).

21. A shining metal M,on burning gives a dazzling white flame and changes to a white 2 Marks
powder N.
(a) Identify M and N.
(b) Represent the above reaction in the form of a balanced chemical equation.

22. Write the functions of the following in the digestive process: 2 Marks
(a) Bile
(b) Bicarbonate secreted by the duodenal wall.
23. Explain giving any three reasons the significance of transpiration in plants. 2 Marks

24. Define peripheral nervous system. What are its components? 2 Marks
25. In the figure given below, a narrow beam of white light is shown to pass through a 2 Marks
triangular glass prism. After passing through the prism, it produces a spectrum XY on
the screen.

(i) Name the phenomenon.


(ii) State the colours seen at X and Y.
(iii) Why do different colours of white light bend at different angles through a prism?

26. What is biological magnification? Will the levels of this magnification be different at 2 Marks
different levels of the ecosystem?

SECTION – C
Section- C consists of 7 question carrying 3 marks each.( 3×7= 21).

27. State the relationship between oxidation and oxidising agent in a redox reaction. 3 Marks
Give an example of redox reaction showing the relationship between oxidation and
oxidising agent.

28. Write balanced chemical equations for the following: 3 Marks


(i) Bleaching powder is kept open in air.
(ii) Blue crystals of copper sulphate are heated.
(iii) Chlorine gas is passed through dry slaked lime.

29. What are the different ways in which glucose is oxidised to provide energy in various 3 Marks
organisms?

30. (i) State the lens's formula and its magnification. 3 Marks
(ii) Using the lens formula, locate the position of an image formed due to an object at
infinity by a convex lens of focal length f.

Or

(i) How will you find the net focal length of a combination of lenses whose focal length
are 15 cm and -5 cm respectively?

31 (a) What is hypermetropia? State the two causes of hypermetropia. 3 Marks


(b) With the help of ray diagrams, show:
(i) The eye-defect hypermetropia.
(ii)Correction of hypermetropia by using a lens.
OR
Draw a labelled diagram of human eye and explain the image formation.
32. Explain the effect on the magnetic field produced at a point in a current carrying 3 Marks
circular coil due to:
(i) increase in the amount of current flowing through it.
(ii) increase in the distance of point from the coil.
(iii) increase in the number of turns of the coil.
33. (a) Which compounds are responsible for the depletion of ozone layer? 3 Marks
(b) Damage to ozone layer is a cause of concern. Justify this statement and suggest two
steps to limit this damage.

SECTION- D
Section-D consists of 3 question carrying 5 marks each.( 3×5= 15).

34. Describe briefly the following chemical properties of carbon compounds and give 5 marks
the chemical equation for each.
1.Combustion
2.Oxidation
3.Addition
4.Substitution
5.Sponification

35. Describe the human female reproductive system with the help of a labelled diagram. 5 Marks
OR

What is meant by tissue culture? How this technique is performed? In which area this
technique is finding its application?

36. State Ohm’s law. Draw a labelled circuit diagram to verify this law in the laboratory. If 5 Marks
you draw a graph between the potential difference and current flowing through a metallic
conductor, what kind of curve will you get? Explain how would you use this graph to
determine the resistance of the conductor.

SECTION- E
Section- E consists of 3 CCT based question carrying 4 marks each.( 4×3= 12).

37. Non-metals are highly electronegative in nature. They tend to gain electrons in their 4 Marks
valence shell to achieve nearest noble gas configuration. Thus, they form anions and
act as good oxidising
agents.
X + H [ ---------------+ X11
(non-metal atom) (anion)

They react with air or oxygen on heating to form oxides which react with water to form
acids. Thus, non- metal oxides are acidic in nature. on-metals do not react with dilute
acids at all. This is because they are electronegative and therefore, cannot displace
hydrogen from acids but they form covalent hydrides when heated with hydrogen.
a). Generally, non-metals are not conductors of electricity. Name One non- metal
which is a good conductor of electricity?

b) What is the Nature of SO2 ?

c) Why are metal reducing agents and non-metals oxidising agents?


OR
Why do nonmetals not conduct electricity?
d) Why are nonmetals electronegative in nature?

38. Mendel crossed a homozygous yellow and round seeded pea plant with another
homozygous pea plant bearing green and wrinkled seeds to obtain the F1 progeny. He
then selfed the progeny of F1 generation to obtain the F2 generation. He † observed that
there were some characters that were expressed in the F2 generation but not in the F1
generation.

a) Write the genotype of the parent pea plants.

b) Write the gametes involved in the formation for F1 progeny.

c) Write down the phenotype of the F1 generation.

d) Write the phenotypic ratio as well as the corresponding phenotype as obtained in the
F2 generation.
OR

What conclusion can be drawn from the fact that some new plants appeared in F2
generation but not in F1 generation.

39. 4 Marks

When the rays of light travels from one transparent medium to another, the path of light
is deviated. This phenomena is called refraction of light. The bending of light depends
on the optical density of medium through which the light pass.
The speed of light varies from medium to medium. A medium in which the speed of light
is more is optically rarer medium whereas in which the speed of light is less is optically
denser medium. Whenever light goes from one medium to another, the frequency of light
does not change however, speed and wavelength change. It concluded that change in
speed of light is the basic cause of refraction.

a) The process of bending of a light ray on changing the medium is named as?
b) A ray of light passes from a medium A to another medium B. No bending of light
occurs if the ray of light hits the boundary of medium B at an angle of?

OR
What is Snell’s law?
c) When light passes from one medium to another, the frequency of light will?

d) When light passes from Rarer to Denser medium, then the speed of light will?
KENDRIYA VIDYALAYA SANGATHAN, JAMMU REGION
SAMPLE PAPER SET 10
CLASS - X SUBJECT : SCIENCE (086)
TIME : 3 hours M. M : 80

General Instructions:
i. This question paper consists of 39 questions in 5 sections.
ii. All questions are compulsory. However, an internal choice is provided in some questions. A
student is expected to attempt only one of these questions.
iii. Section A consists of 20 objective type questions carrying 1 mark each.
iv. Section B consists of 6 Very Short questions carrying 02 marks each. Answers to these
questions should in the range of 30 to 50 words.
v. Section C consists of 7 Short Answer type questions carrying 03 marks each. Answers to these
questions should in the range of 50 to 80 words
vi. Section D consists of 3 Long Answer type questions carrying 05 marks each. Answer to these
questions should be in the range of 80 to 120 words.
vii. Section E consists of 3 source-based/case-based units of assessment of 04 marks each with
sub-parts.
Section A
1 After burning magnesium ribbon, a white powder is formed. What is the nature of this powder
formed?
1
(a) Neutral
(b) Acidic
(c) Basic
(d) None of the above

In the experimental set up given below if the air is too moist, calcium chloride is used. What 1
2 function it does here?
(a) It reacts with HCl
(b) It makes dry HCl moist
(c) It absorbs moisture from air
(d) It is an oxidizing agent
3. When sodium sulphate solution mixed with barium chloride solution it forms a white precipitate.
Which type of reaction is involved in its formation?
(a) Double displacement reaction 1
(b) Redox Reaction
(c) Decomposition reaction
(d) Photochemical reaction

4. What changes will you observe when silver chloride is kept in sunlight? 1
(a) Its colour changes from white to grey due to photochemical reaction
(b) Its colour changes from grey to black due to photochemical reaction
(c) It starts to burn
(d) It does not show any change

5. Which among the following is/are oxidizing agents? 1


(a) Potassium permanganate
(b) Potassium dichromate
(c) Both a & b
(d) None of the above

6. The gas produced when metal reacts with an acid like HCl is:
(a) Carbon Dioxide
(b) Oxygen 1
(c) Hydrogen
(d) Nitrogen
7. The most reactive and least reactive metal respectively among the following are: 1
Pb, Al, K, Mg, Ca, C, Zn, Sn, Au.
(a) Zn & Al
(b) Mg & C
(c) K & Sn
(d) K & Au
8. In the following reaction what is the role of concentrated Sulphuric acid?
(a) It acts as oxidizing agent
(b) It acts as reducing agent 1
(c) It acts as hydrating agent
(d) It acts as dehydrating agent

9. Which statement about arteries is not true: 1


(a) They always carry blood from heart to body parts
(b) They carry blood under high pressure
(c) They have thick and elastic walls
(d) They always carry oxygenated blood
10. Find the odd one out?
(a) Auxin
(b) Cytokinin 1
(c) Insulin
(d) Abscisic Acid

11. In flowering plants or angiosperms, the ovule, after fertilization, develops into a:
(a) Fruit 1
(b) Flower
(c) Seed
(d) New plant
12. If a pea plant having round seeds is crossed with a pea plant having wrinkled seeds, then, what
will be the percentage of plants in F2 generation that will have wrinkled seeds?
(a) 50% 1
(b) 75%
(c) 25%
(d) 100%

13. In a circuit if the connecting wires, made of copper, are replaced by wires made of an alloy, the
resistance of the circuit will:
(a) Decrease 1
(b) Increase
(c) Will remain same
(d) None of the above
14. The hinderance presented by material of the conductor to the smooth passing of electric current is
known as : 1
(a) Resistance
(b) Conductance
(c) Inductance
(d) None of the above
15. Imagine electrons moving in a direction going into the plane of the paper. The direction of the
magnetic field lines as seen from above will be:
(a) Anti-clockwise 1
(b) Clockwise
(c) Upwards
(d) Downwards
16. In a magnetic field, the magnetic field line shows the path that will be followed by a:
(a) North pole
(b) South pole 1
(c) Current
(d) None of the above
Q. No 17 to 20 are Assertion - Reasoning based questions.
These consist of two statements – Assertion (A) and Reason (R). Answer these questions
selecting the appropriate option given below:
(a) Both A and R are true and R is the correct explanation of A
(b) Both A and R are true and R is not the correct explanation of A
(c) A is true but R is false
(d) A is False but R is true

17. Assertion: Amoeba exhibits holozoic mode of nutrition.


Reason: Decomposers exhibit saprotrophic mode of nutrition. 1
18. Assertion: Blood from the atria moves into the ventricles through valves.
Reason: Valves prevent back-flow of blood. 1
19. Assertion: Tall and dwarf pea plants were crossed and the F1 generation had half tall and half
dwarf plants. 1
Reason: The parental generation consisted of heterozygous tall and homozygous dwarf pea plants.
20. Assertion: Magnetic field strength is inversely proportional to the current flowing in the coil.
Reason: Magnetic field strength is directly proportional to the number of turns in a solenoid. 1
Section B

21. A student observes the reaction of dilute Nitric acid with four metals and records his observations
in the table below. Match the metals with the correct products that will be formed:

Metal One of the Product formed


Iron H2O
Magnesium H2
Copper H2
Manganese H2

OR 2

Kishan, while experimenting with metals saw that two metals reacted violently with cold water
whereas the reaction of another metal was less violent. Another metal reacts with hot water and
starts floating. Identify metals in each case.

22. How do plants manage to transport water to great heights? 2

23. Can glomerular filtration in nephrons alone suffice the function of blood filtration or excretion? 2

24. What are reflex actions? Give an example. 2

25. In the diagram shown below, write the names of the phenomena involved in rainbow formation.

2
OR

How does a normal human eye adjust itself to look clearly at nearer and objects?

26. What is biomagnification? Give an example. 2

Section-C

27. Consider the following reaction of an ionic compound AB:


3
X + AB AX + B

Identify what has been oxidized to what & which species has been reduced. Also, name oxidizing &
reducing agents.

28. What is chlor-alkali process? 3


What are the products formed during the process and what are their uses?
29. In the experimental set up below what role does potassium hydroxide play.
If jar X is also kept without pottasium hydroxide, how will it affect the conclusion of the
test/experiment?

OR

The structure of the blood vessels gives us an idea about how and what composition of blood flows
through them. Explain.
30. You want to project the image of a candle flame on a screen using a magnifying lens of focal
length 20 cm.
(i) At what distance from the lens you should place the candle to get such an image? 3
(ii) Draw a ray diagram to show the image formation.
(iii) What will be the nature and size of image formed if the candle is placed 5 cm in front
of the lens?

31. A girl is using lenses with power -4 D to see like a normal person.
(i) What type of eye defect she is suffering from? What are its possible causes? 3
(ii) If the girl wrongly uses a lens of with power +4 D, what will happen to the near point
of her eye?

32. PQ is a current carrying conductor in the plane of the paper as shown in the figure. Mention the
direction of magnetic fields produced by it at points S and R. Given r2 < r1, where will the strength
of the magnetic field be greater? 3

OR
Explain the effect on the magnetic field produced at a point in a current carrying circular coil due
to:
(i) increase in the amount of current flowing through it
(ii) increase in the distance from the coil
(iii) increase in the number of turns of the coil.
33. What are trophic levels? What limits the number of trophic levels in a food chain? 3

Section-D

34. Akash added some amout of alkaline potassium permanganate to warm ethanol in a test tube. He
saw colour of potassium permanganate go away along with pungent smell coming from the test
tube caused due to formation of compound A. This compound A reacts with sodium bicarbonate
to form a gas B.
(a) Identify compounds A & B.
(b) Write the chemical reaction involved in the formation of compound A.
(c) What is the role of alkaline potassium permanganate in the reaction.
5
OR

Varsha while testing two hydrocarbons A & B found that A burns with a yellow and sooty flame
whereas B burns with a clean blue flame.
(a) What type of compounds are A & B?
(b) Describe what will happen if A & B are made to react with hydrogen. Write the chemical
reactions involved.
(c) B reacts with compound C in presence of sunlight to undergo a substitution reaction
formimg HCl. Identify C.
35. (a) What happens when a Planaria gets cut into many pieces? What is the mode of
reproduction in planaria called?
(b) The species reproducing sexually has comparatively better chances of survival than the one
reproducing asexually? Justify this statement. 5

OR

(a) Describe the process that follows after the landing of pollen grains on stigma during
fertilization in plants?

(b) List the parts of human male reproductive system which contribute fluid to the semen.
State two advantages semen offers to the sperms.
36. (a) Why should a fuse with defined rating for an electric circuit not be replaced by one with a
larger rating?
(b) An electrical fuse is rated at 2 A. What is meant by this statement? 5
(c) An electric iron of 1 kW is operated at 220 V. Find which of the following fuses that
respectively rated at 1 A,3 A and 5 A can be used in it.

SECTION - E
Q.no. 37 to 39 are case - based/data -based questions with 2 to 3 short sub - parts. Internal choice
is provided in one of these sub-parts.
37. Anamika was testing the reactions exhibited by four metals A,B,C & D. She found that metal B &
A replace C from its salt in solution. A can replace C but not B whereas D can replace B from its
salt solution.
(a) From the information given above, write the names of the four metals in the decreasing
order of their reactivity.
(b) What will happen if a salt of metal A in solution reacts with metal C? Justify your answer. 4

OR
(b) In another experiment Anamika found that a low reactive metal E when heated in air forms
a black oxide of the metal which is basic in nature.
Identify the metal E and write the chemical reaction involved.
38. Traits run in families from one generation to another. Hair colour, texture, skin colour , etc are
some traits to mention. Gaurav has 2 brothers and one sister out of whom only Gaurav has brown
hair like his father whereas others in his family like her mother have black hair.
(a) From the above information can we tell that brown hair colour is a recessive trait?
(b) Does the inheritance of hair colour depend on the sex? 4
(c) With help of a flowchart show the genotype determining hair colour of each family
member?

OR
(d) If Gaurav marries Kajal who has black hair colour, what is the probability of their children
having black hair?
39. A security person at a shopping mall was looking a mirror and saw saw image of his friend in the
mirror which was always erect whatever be the location of his friend. At a particular moment the
image was five times lesser in size than the actual size. At the time his friend was 500 cms away
from the mirror.
(a) What type of mirror has been used in this situation? Justify your answer.
(b) What are the uses of the type of mirror used in the mall?
(c) What is the focal length of the mirror? 4
Or
(c) Would the security guard be able to see similar type of images of his friend if some other
type of mirror is used?
KENDRIYA VIDYALAYA SANGATHAN, JAMMU REGION

SAMPLE PAPER SET 11

CLASS - X SUBJECT : SCIENCE (086)

TIME : 3 hours M. M : 80

General Instructions:

i. This question paper consists of 39 questions in 5 sections.


ii. All questions are compulsory. However, an internal choice is provided in some
questions. A student is expected to attempt only one of these questions.
iii. Section A consists of 20 objective type questions carrying 1 mark each.
iv. Section B consists of 6 Very Short questions carrying 02 marks each. Answers to
these questions should in the range of 30 to 50 words.
v. Section C consists of 7 Short Answer type questions carrying 03 marks each.
Answers to thesequestions should in the range of 50 to 80 words
vi. Section D consists of 3 Long Answer type questions carrying 05 marks each.
Answer to thesequestions should be in the range of 80 to 120 words.
vii. Section E consists of 3 source-based/case-based units of assessment of 04 marks each
with sub-parts.

SECTION-A
Select and write one most appropriate option out of the four options given for each of the
questions 1 – 20
Q.No. Questions Marks

1 Which of the following statements about the reaction below are incorrect? 1
2PbO (s) + C (s) 2Pb (s) + CO2
a) Lead is getting reduced
b) Carbon dioxide is getting oxidized
c) Carbon is getting oxidized
d) Lead oxide is getting reduced
i. a and b
ii. a and c
iii. a, b, and c
iv. all

2 Fe2O3 + 2Al Al2O3 + 2Fe 1


The above reaction is an example of a
a) Combination reaction
b) Double displacement reaction
c) Decomposition reaction
d) Displacement reaction

3 Name the compound which on treatment with chlorine yields bleaching 1


powder.
a. Calcium oxide
b. Calcium hydroxide
c. Calcium sulphide
d. All
4 What is the common name of Na2CO3.10H2O? 1
a. Washing soda
b. Baking soda
c. Baking powder
d. Both b and c

5 A non-metal X exists in two different forms Y and Z. Y is the hardest natural 1


substance, whereas Z is a good conductor of electricity. Identify X, Y and Z.
a. Carbon, graphite, diamond
b. Graphite, carbon and diamond
c. Carbon, diamond, graphite
d. Diamond, carbon, graphite

6 Some materials are tested on a litmus paper to test the action. The materials are
Dry HCl gas, lemon juice, curd, soap solution. Which of these will change blue
litmus to red
a. Dry HCl gas and Lemon juice
b. Lemon juice and curd
c. Curd and soap solution
d. All will convert blue to red
7 Ethane with molecular formula C2H6 has 1
a) 6 covalent bonds
b) 7 covalent bonds
c) 8 covalent bonds
d) 9 covalent bonds

8 What will happen if platelets were absent in the blood? 1


a. Blood will become decoloured
b. The blood will become thick
c. No clotting of wound
d. It will affect Hemoglobin

9 CO2 is essential for the process of photosynthesis. This process was 1


demonstrated by Mohl half leaf experiment. The chemical used by Mohl to
absorb carbon dioxide was
a. Ammonia (NH3)
b. Nitrogen di oxide (NO2)
c. Potassium hydroxide (KOH)
d. Calcium oxide (CaO)

10 A Mendalian experiment consisted of breeding tall pea plants bearing violet 1


flowers with short pea plants bearing white flowers. The progeny all bore
violet flowers, but almost half of them were short. This suggests that the
genetic makeup of all the tall parent can be depicted as:
a) TTWW
b) TTww
c) TtWW
d) TtWw

11 When plants move towards source of gravity it is called as 1


a. Positive geotropism
b. Negative geotropism
c. Phototropism
d. Hydrotropism

12 During double fertilization in plants one male sperm fused with egg and the 1
second fuses with

a. Antipodal nucleus
b. Central cell nucleus
c. Synergid cell nucleus
d. None
13 In a series combination of resistors, what remains constant- current or voltage? 1
a. Current
b. Voltage
c. Both
d. None
14 List two special characteristics of a heating element wire. 1
a. High melting point and low resistivity
b. High melting point and High resistivity
c. Low melting point high resistivity
d. Low melting point low resistivity
15 At the time of short circuit, the current in the circuit: 1
a) Reduces substantially
b) Does not change
c) Increases heavily
d) Varies continuously
16 What is the potential difference maintained between the live wire and the 1
neutral wire in the domestic circuits in India?
a) 110 V
b) 160 V
c) 220 V
d) 440 V
Questions 17-20 consist of two statements – Assertion (A) and Reason (R).
Answer these questions selecting the appropriate option given below:
(a) Both A and R are true and R is the correct explanation of A.
(b) Both A and R are true but R is not the correct explanation of A.
(c) A is true but R is false.
(d) A is false but R is true.

17 Assertion (A) : Decomposition of vegetable matter into compost is an 1


example of exothermic reactions.
Reason (R) : Exothermic reaction are those reactions in which heat is
evolved.
18 Assertion(A) : Variations are seen in offspring produced by sexual 1
reproduction.
Reason (R) : DNA molecule generated by replication is not exactly
identical to original DNA.
19 Assertion (A) : Bile is essential for digestion of lipids. 1
Reason (R) : Bile juice contains enzymes.
20 Assertion(A): The magnitude of the magnetic field at a point on the 1
axis of a current carrying solenoid is inversely proportional to the
current flowing through the solenoid.
Reason (R) : The magnitude of the magnetic field at a point on the axis
of a current carrying solenoid is directly proportional to the number of
turns per unit length of a solenoid.
SECTION-B
Q. No. 21 to 26 are very short answer questions.
21 In a solution of CuSO4 was kept in an iron pot. After few days the iron pot was 2
found to have a number of holes in it. Explain the reason in terms of reactivity.
Also write the equation of the reaction involved.
OR
Why do silver articles turn black when kept in open for few days? Name the
phenomenon involved.

22 Mention the ways in which the digestive system of herbivores and carnivores is 2
different?
Or
Mention the ways in which is digested food is utilized when it reaches the body
cells.

23 What is the use of iodised salt available? 2


24 Why are no valves found in the arteries? 2
25 Why the sky looks dark instead of blue to an astronaut? 2
OR
The far point of a myopic person is 80 cm in front of eye. What is the nature
and power of lens required to correct the problem?

26 What will happen if we kill all the organisms in one trophic level? 2

SECTION - C
Q.No. 27 to 33 are short answer questions.
27 What is precipitation reaction? Explain by giving examples. 3
28 The pH values of the solutions P,Q, R, S and T are 3,1,12, 7 and 10 3
respectively.
a) Arrange these solutions in the decreasing order of pH values.
b) Arrange these solutions in the decreasing order of hydrogen ion
concentration.
c) In the list of the given solutions identify:
i. Neutral solutions
ii. Highly alkaline solution
iii. Highly acidic solution
iv. Less acidic solution
v. Less basic solution

29 Draw a labelled diagram to show the various parts of human excretory 3


system. Briefly state the structure and function of each part.
OR
Leaves of a healthy potted plant were coated with Vaseline. Will this
plant remain healthy for long? Give reasons for your answer.
30 A Convex lens forms a real and inverted image of a needle at a distance of 50 3
cm from it. Where is the needle placed in front of the convex lens if the image
is equal to the size of the object? Also find the power of the lens.
31 An object is placed at a distance of 10 cm from a convex mirror of focal 3
length 15 cm. find the position and nature of the image.
32 (a) Draw magnetic field lines produced around a current carrying straight 3
conductor passing through cardboard. How will the strength of the
magnetic field change, when the point where magnetic field is to be
determined, is moved away from the straight wire carrying constant
current? Justify your answer.
(b) Two circular coils A and B are placed close to each other. If the current
in the coil A is changed, will some current be induced in the coil B? Give
reason.
OR
(a) Shruti draws magnetic field lines close to the axis of a current carrying
circular loop. As she moves away from the centre of circular loop, she
observes that the lines keep on diverging. Explain the reason for her
observation.
(b) Write two properties of magnetic field lines.

33 What are the by-products of fertilizer industries? How do they affect 3


the environment?

SECTION - D
Q.No. 34 to 36 are Long answer questions.
34 (a) What are hydrocarbons? Give examples. 5
(b) Give the structural differences between saturated and unsaturated
hydrocarbons with two examples each.
(c) What is a functional group? Give examples of four different
functional groups.

OR
a) Write the formula and draw the electron dot structure of carbon
tetrachloride.
b) What is saponification? Write the reaction involved in this process.

35 A) Describe the structure and function of placenta. 5


B) write the function of the following parts of human female reproductive
system:
i. Ovary
ii. Oviduct
iii. Uterus
OR
a. What is DNA copying? State its importance.
b. List four advantages of vegetative propagation

36 A) Three resistors of 5Ω, 10 Ω and 15 Ω are connected in series and the 5


combination is connected to the battery of 30 V. ammeter and voltmeter are
also connected in the circuit. Draw a circuit diagram to connect all the devices
in proper correct order. Find the current flowing and potential difference across
10 Ω resistor.

B) An electric bulb is connected to a 200 V generator. The current is


0.5 A. what is the power of the bulb?
C) Define electric power and also write its unit.

OR
Two identical resistors each of resistance 10 Ω are connected in
i) series and
ii) Parallel to a 6 V battery. Calculate the ration of power
consumed in the combination of resistors in two cases.

SECTION - E
Q.No. 37 to 39 are case - based/data -based questions with 2 to 3 short sub - parts. Internal
choice is provided in one of these sub-parts
37 The metals in the middle of the activity series such as iron, zinc, lead, copper, 4
etc., are moderately reactive. These are usually present as sulphides or
carbonates in nature. It is easier to obtain a metal from its oxide, as compared
to its sulphides and carbonates. Therefore, prior to reduction, the metal
sulphides and carbonates must be converted into metal oxides. The sulphide
ores are converted into oxides by heating strongly in the presence of excess air.
This process is known as roasting. The carbonate ores are changed into oxides
by heating strongly in limited air. This process is known as calcination.
i) What is Calcination?
ii) Give a suitable example for the process Roasting.
iii) What is the process of converting metal oxide to metal?
OR
Name some other reducing agent which can be used to extract metal from
metal oxide?

38 Read the following and answer any four questions from (i) to (iv). 4

Pea plants can have smooth seeds or wrinkled seeds. One of the
phenotypes is completely dominant over the other. A farmer decides to
pollinate one flower of a plant with smooth seeds using pollen from
plant with wrinkled seeds. The resulting pea pod has all smooth seeds.

(i) Which of the following conclusions can be drawn?


(1) The allele for smooth seeds is dominated over that of wrinkled
seeds.
(2) The plant with smooth seeds is heterozygous.
(3) The plant with wrinkled seeds is homozygous.
(a) 1 only
(b) 1 and 2 only
(c) 1 and 3 only
(d) 1, 2 and 3

(ii) Which of the following crosses will give smooth and wrinkled
seeds in same proportion?
(a) RR X rr
(b) Rr x rr
(C) RRX Rr
(d) rrrr

(iii) Which of the following cross can be used to determine the


genotype of a plant with dominant phenotype?
(a) RR RR
(b) Rr x Rr
(c) Rr RR
(d) RR x rr

OR

On crossing of two heterozygous smooth seeded plants (Rr), a total of 1000


plants were obtained in F1 generation. What will be the respective number
of smooth and wrinkled seeds obtained in F1 generation?

39 Read the following and answer any four questions from (i) to (iv). 4

The spherical mirror forms different types of images when the object is
placed at different locations. When the image is formed on screen, the
image is real and when the image does not form on screen, the image is
virtual. When the two reflected rays meet actually, the image is real and
when they appear to meet, the image is virtual.
A concave mirror always forms a real and inverted image for different
positions of the object. But if the object is placed between the focus and
pole. the image formed is virtual and erect.
A convex mirror always forms a virtual, erect and diminished image. A
concave mirror is used as doctor’s head mirror to focus light on body
parts like eyes, ears, nose etc., to be examined because it can form erect
and magnified image of the object. The convex mirror is used as a rear
view mirrors in automobiles because it can form an small and erect
image of an object.

(i) When an object is placed at the center of curvature of a concave


mirror, what will be size of the image?

(ii) No matter how far you stand from a mirror, your image appears
erect. What type of mirror it could be?
(iii) A child is standing in front of a magic mirror. She finds the
image of her head bigger, the middle portion of her body of the
same size and that of the legs smaller. What is the order of
combinations for the magic mirror from the top?

OR

To get an image larger than the object, one can use which type of
mirror?
KENDRIYA VIDYALAYA SANGATHAN, JAMMU REGION
SAMPLE PAPER SET 12
CLASS - X SUBJECT : SCIENCE (086)
TIME : 3 hours M. M : 80

General Instructions:
i. This question paper consists of 39 questions in 5 sections.
ii. All questions are compulsory. However, an internal choice is provided in some questions. A
student is expected to attempt only one of these questions.
iii. Section A consists of 20 objective type questions carrying 1 mark each.
iv. Section B consists of 6 Very Short questions carrying 02 marks each. Answers to these
questions should in the range of 30 to 50 words.
v. Section C consists of 7 Short Answer type questions carrying 03 marks each. Answers to these
questions should in the range of 50 to 80 words
vi. Section D consists of 3 Long Answer type questions carrying 05 marks each. Answer to these
questions should be in the range of 80 to 120 words.
vii. Section E consists of 3 source-based/case-based units of assessment of 04 marks each with
sub-parts.
SECTION - A
Select and write one most appropriate option out of the four options given for each of the
questions 1 – 20

Q.1 Which of the following is a displacement reaction?

Q.2 In terms of acidic strength, which one of the following is in the correct increasing order?
(a) Water < Acetic acid < Hydrochloric acid
(b) Water < Hydrochloric acid < Acetic acid
(c) Acetic acid < Water < Hydrochloric acid
(d) Hydrochloric acid < Water < Acetic acid

Q. 3 What is formed when zinc reacts with sodium hydroxide?


(a) Zinc hydroxide and sodium
(b) Sodium zincate and hydrogen gas
(c) Sodium zinc-oxide and hydrogen gas
(d) Sodium zincate and water
Q.4 Which property of metals is used for making bells and strings of musical instruments like
Sitar and Violin?
(a) Malleability
(b) Sonorousness
(c) Ductility
(d) Conductivity
Q.5 The image represents the structure of a few hydrocarbon compounds.

Which of these compounds can be classified as alkynes?


(a) only (A)
(b) only (B)
(c) both (A) and (D)
(d) both (B) and (C)
Q. 6 What are the products obtained by anaerobic respiration in plants?
(a) Lactic acid + Energy
(b) Carbon dioxide + Water + Energy
(c) Ethanol + Carbon dioxide + Energy
(d) Pyruvate
Q.7 Breathing is controlled by which part of the brain?
(a) Cerebrum
(b) Cerebellum
(c) Hypothalamus
(d) Medulla oblongata
Q.8 The secretion of which hormone leads to physical changes in the body when you are 10-12
years of age?
(a) estrogen from testes and testosterone from ovary.
(b) Estrogen from adrenal gland and testosterone from pituitary gland.
(c) Testosterone from testes and estrogen from ovary.
(d) Testosterone from thyroid gland and estrogen from pituitary gland.
Q. 9 The correct sequence of organs in the male reproductive system for transport of
sperms is
a) Testis → vas deferens → urethra
b) Testis → ureter → urethra
c) Testis → urethra → ureter
d) Testis → vas deferens → ureter
Q.10 What can be concluded about the division in plasmodium?
a) The cyst divides repeatedly to form many daughter cells.
b) The cell divides multiple times giving rise to many daughter cells.
c) The nucleus divides repeatedly inside the cell to form new daughter cells.
d) The cyst enlarges in size and then bursts producing many new daughter cells.
Q.11 A student studies that the speed of light in air is 300000 kms/ sec where that of speed
in a glass slab is about 197000 kms/ sec. What causes the difference in speed of light in
these two media?
(a) difference in density
(b) difference in temperature
(c) difference in amount of light
(d) difference in direction of wind flow

Q.12 Which of the following is the cause of soil pollution?

a) Ozone
b) Aerosol
c) Acid rain
d) None of the above

Q.13 The splitting of white light into different colours on passing through a prism is called
(a) reflection
(b) refraction
(c) dispersion
(d) deviation
Q.14 The resistance whose V – I graph is given below is
(a) 5/3 Ω
(b) 3/5 Ω
(c) 5/2 Ω
(d) 2/5 Ω
Q.15 The image shows a combination of 4 resistors.

What is the net resistance between the two points in the circuit?
(a) 0.5 Ω
(b) 1.0 Ω
(c) 1.5 Ω
(d) 2.0 Ω

Q.16 Which of the following is the main reason for producing the atmospheric greenhouse effect?

a. Absorption and re-emission of ultraviolet radiations by the atmosphere

b. Absorption and re-emission of infrared radiations by the atmosphere


c. Absorption and re-emission of visible light by the atmosphere
d. None of the above

Q. no 17 to 20 are Assertion - Reasoning based questions.


These consist of two statements – Assertion (A) and Reason (R). Answer these questions
selecting the appropriate option given below:
(a) Both A and R are true and R is the correct explanation of A
(b) Both A and R are true and R is not the correct explanation of A
(c) A is true but R is false
(d) A is False but R is true
Q.17 Assertion (A) : Calcium carbonate when heated gives calcium oxide and water.
Reason (R) : On heating calcium carbonate, decomposition reaction takes place.
Q.18 Assertion: Vegetable oils are unsaturated, react with hydrogen in presence of nickel to
form vegetable ghee.
Reason: This reaction is saponification.
Q.19 Assertion : Phloem helps in translocation of food from the leaves.
Reason: Phloem provides mechanical support to plant.
Q.20 Assertion : Accumulation of variation in a species increases the chances of its survival in
changing environment.
Reason : Accumulation of heat resistance in some bacteria ensure their survival even when
temperature in
environment rises too much.

SECTION B
Q.21 . Name the products formed when iron filings are heated with dilute hydrochloric acid.
Q.22 The electronic configurations of three elements X, Y and Z are X — 2, 8; Y — 2, 8, 7 and
Z — 2, 8, 2. Which of the following is A metal, non metal, alloys.
Q.23 What are the different modes of asexual reproduction?
Q.24 What are the components of the chromosome?
Q.25 Why does the objective lens of an astronomical telescope have a large light gathering
power?
OR
How does the eye adjust itself to deal with light of varying intensity?
Q.26 What are the after effects of ozone depletion?

OR
List out the components in a food chain?

SECTION C
Q.27 Complete the missing components/variables given as x and y in the following reactions:

(a) Pb (NO3)2 (aq) + 2KI (aq) → Pbl2 (x) + 2KNO3 (y)


(b) Cu (s) + 2AgNO3 (aq) → Cu (NO3)2 (aq) + x (s)
(C ) Na2SO4 + BaCl2 → ……(X)……. + 2NaCl

Q.28 What is observed when carbon dioxide gas is passed through lime water
(i) for a short duration?
(ii) for a long duration? Also write the chemical equations for the reactions involved..
Q.29 Draw a diagram of human respiratory system and label – pharynx, trachea, lungs,
diaphragm and alveolar sac on it.
Q.30 Reflex arcs continue to be more efficient for quick responses”. Justify this statement giving
reason.
Q.31 a) What is spore formation?
(b) Draw a diagram showing spore formation in Rhizopus.
(c) List two advantages for organisms to reproduce themselves through spores.

OR

Mention the methods used for regulation of childbirth.

Q. 32 Draw a labelled diagram of structure of eye.

Q.33 A convex lens has a focal length of 10cm. At what distance from the lens should the object
be placed so that it forms a real and inverted image 2020 cm. away from the lens? What would
be the size of the image formed if the object is 22 cm high? With the help of a ray, the diagram
shows the formation of the image by the lens in this case?
OR
A convex lens forms a real and inverted image of a needle at a distance of 50 cm. from it. Where
is the needle placed in front of the convex lens if the image is equal to the size of objects? Also,
find the power of the lens.
SECTION D
Q.34 A compound X is formed by the reaction of carboxylic acid C2H4O2 and alcohol in the
presence of a few drops of H2SO4. The alcohol on oxidation with alkaline KMnO4 followed
by acidification gives the same carboxylic acid as used in this reaction. Give the names and
structures of (a) carboxylic acid, (b) alcohol and (c) compound X. Also, write the reaction.
Q.35 a) Why did Mendel carry out an experiment to study inheritance of two traits in garden
pea?
(b) What were his findings with respect to inheritance of traits in F1 and F2 generation?
(c) State the ratio obtained in the F2 generation in the above mentioned experiment.
OR
A green stemmed rose plant denoted by GG and a brown stemmed rose plant denoted by gg are
allowed to undergo a cross with each other.
(a) List your observations regarding:
(i) Colour of stem in their F1 progeny
(ii) Percentage of brown stemmed plants in F2 progeny if plants are self-pollinated.
(iii) Ratio of GG and Gg in the F2 progeny.
(b) Based on the findings of this cross, what conclusion can be drawn?
Q.36 (i) List three factors on which the resistance of a conductor depends.

(ii) Calculate the resistance of a metal wire of length 2m and area of cross section 1.55 × 106 m²,
if the resistivity of the metal be 2.8 × 10-8 Ωm.

OR

(i) Define the resistivity.

(ii) A bulb is rated 40 W; 220 V. Find the current drawn by it, when it is connected to a 220 V
supply. Also find its resistance. If the given bulb is replaced by a bulb of rating 25 W; 220 V,
will there be any change in the value of current and resistance? Justify your answer and
determine the change.
SECTION E
Q.37 The metals in the middle of the activity series such as iron, zinc, lead, copper, etc., are
moderately reactive. These are usually present as sulphides or carbonates in nature. It is easier to
obtain a metal from its oxide, as compared to its sulphides and carbonates. Therefore, prior to
reduction, the metal sulphides and carbonates must be converted into metal oxides. The sulphide
ores are converted into oxides by heating strongly in the presence of excess air. This process is
known as roasting. The carbonate ores are changed into oxides by heating strongly in limited air.
This process is known as calcination

i) What is Calcination?
ii) Give a suitable example for the process Roasting.
iii) What is the process of converting metal oxide to metal?
0R
Name some other reducing agent which can be used to extract metal from metal oxide?

Q.38 Heterotrophic nutrition is a mode of nutrition in which organisms obtain readymade


organic food from outside sources. The organisms that depend upon outside sources for
obtaining organic nutrients are called heterotrophs.
Heterotrophic nutrition is of three types: saprophytic, parasitic and holozoic nutrition.

(i) In which of the following groups of organisms food material is broken outside the body
and absorbed?
(a) Mushroom, green plants, Amoeba
(b) Yeast, mushroom, bread mould
(c) Paramecium, Amoeba, Cuscuta
(d) Cuscuta, lice, tapeworm

(ii) Which of the following is a parasite?


(a) Yeast
(b) Taenia
(c) Amoeba
(d) Earthworm

(iii) Define the saprotroph with example.


OR

Define the heterotrophic nutrition with example.

Q.39 The lenses forms different types of images when object placed at different locations. When
a ray is incident parallel to the principal axis, then after refraction, it passes through the focus or
appears to come from the focus.

When a ray goes through the optical centre of the lens, it passes without any deviation. If the
object is placed between focus and optical center of the convex lens, erect and magnified image
is formed.

As the object is brought closer to the convex lens from infinity to focus, the image moves away
from the convex lens from focus to infinity. Also the size of image goes on increasing and the
image is always real and inverted.

A concave lens always gives a virtual, erect and diminished image irrespective to the position of
the object.

(i) The location of image formed by a convex lens when the object is placed at
infinity is
(a) at focus
(b) at 2F
(c) at optical center
(d) between Fand 2F

(ii) (ii) When the object is placed at the focus of concave lens, the image formed is
(a) real and smaller
(b) virtual and inverted
(c) virtual and smaller
(d) real and erect
(iii) Draw a ray diagram of image formed by a convex lens when the object is placed at
the focus F1 of convex lens
OR

Draw a ray diagram of image formed by a convex lens when the object is placed at 2F2 in
front of convex lens, the location of image .
KENDRIYA VIDYALAYA SANGATHAN, JAMMU REGION
SAMPLE PAPER SET 13
CLASS - X SUBJECT : SCIENCE (086)
TIME : 3 hours M. M : 80

General Instructions :
(i) The question paper comprises four sections A, B, C and D. There are 38 questions
in the question paper.
(ii) All questions are compulsory.
(iii) Section–A – question no. 1 to 20 - all questions and parts thereof are of one mark
each. These questions contain multiple choice questions (MCQs), very short
answer questions and assertion - reason type questions. Answers to these should
be given in one word or one sentence.
(iv) Section–B – question no. 21 to 26 are short answer type questions, carrying 2
marks each. Answers to these questions should in the range of 30 to 50 words.
(v) Section–C – question no. 27 to 33 are short answer type questions, carrying 3
marks each. Answers to these questions should in the range of 50 to 80 words.
(vi) Section–D – question no. 34 to 36 long answer type questions carrying 5 marks
each. Answer to long answer type questions should be in the range of 80 to 120
words. and
(vii) Section–E – question no. 37 to 39 are CCT base question
(viii) There is no overall choice. However, internal choices have been provided in some
questions. A student has to attempt only one of the alternatives in such questions.
(ix) Wherever necessary, neat and properly labelled diagrams should be drawn

Section A
Q1 The following reaction is an example of a 4NH3(g) + 5O2(g) → 4NO(g) + 6H2O(g)
(i) displacement reaction
(ii) combination reaction
(iii) redox reaction
(iv) neutralisation reaction
(a) (i) and (iv)
(b) (ii) and (iii)
(c) (i) and (iii)
(d) (iii) and (iv)
Q2 Three beakers labelled as A, B and C each containing 25 mL of water were taken. A small
amount of NaOH, anhydrous CuSO4 and NaCl were added to the beakers A, B and C
respectively. It was observed that there was an increase in the temperature of the solutions
contained in beakers A and B, whereas in case of beaker C, the temperature of the solution
falls. Which one of the following statement(s) is(are) correct?
(i) In beakers A and B, exothermic process has occurred.
(ii) In beakers A and B, endothermic process has occurred.
(iii) In beaker C exothermic process has occurred.
(iv) In beaker C endothermic process has occurred.
(a) (i) only
(b) (ii) only
(c) (i) and (iv)
(d) (ii) and (iii)
Q3 When green coloured ferrous sulphate crystals are heated, the colour of the crystal
changes because
(a) it is decomposed to ferric oxide
(b) it loses water of crystallisation
(c) it forms SO2
(d) it forms SO3
Q4 Sodium hydrogencarbonate when added to acetic acid evolves a gas.
Which of the following statements are true about the gas evolved?
(i) It turns lime water milky
(ii) It extinguishes a burning splinter
(iii) It dissolves in a solution of sodium hydroxide
(iv) It has a pungent odour
(a) (i) and (ii) (b) (i), (ii) and (iii)
(c) (ii), (iii) and (iv) (d) (i) and (iv)

Q5 In an attempt to demonstrate electrical conductivity through an electrolyte, the following

apparatus (Figure ) was set up.


Which among the following statement(s) is(are) correct?
(i) Bulb will not glow because electrolyte is not acidic
(ii) Bulb will glow because NaOH is a strong base and furnishes ions for conduction.
(iii) Bulb will not glow because circuit is incomplete
(iv) Bulb will not glow because it depends upon the type of electrolytic solution
(a) (i) and (iii) (b) (ii) and (iv)
(c) (ii) only (c) (iv) only

Q6 In the given reaction, Al2O3 + NaOH → ……X…… + H2O


What is element X?
a) NaAlO2
b) Na3Al
c) Na2O3
d) NaAl2O3

Q7 The structural formula of ethyl ethanoate

Q8 A student sets up an experiment to study the role of enzymes in digestion of food.

In which test tube, the digestion of protein will occur?


(a) Test tubes A as pepsin will breakdown protein into simple molecules.
(b) Test tube B as HCl will breakdown protein into simple molecules.
(c) Test tube A as pepsin will breakdown into simple molecules.
(d) Test tube B as HCl will activate pepsin for breakdown of protein into simple molecules

Q9 A student sets up an experiment to study human respiration using lime water, test tube
and a straw. Lime water is colourless in the absence of CO2 and is milky in its presence. The
student fills a freshly prepared limewater in a test tube and blows air through straw into the
limewater. It was observed that the solution turns cloudy as shown. What can be evaluated
from this observation?
(a) Oxygen is exhaled during respiration.
(b) Water vapours are produced during respiration.
(c) Carbon dioxide is exhaled during respiration.
(d) Glucose is produced during respiration.

Q10 Which is the correct sequence of the components of a reflex arc?


(a) Receptors→ Muscles→ Sensory neuron→ Motor neuron→ Spinal cord
(b) Receptors→ Motor neuron → Spinal cord → Sensory neuron → Muscle
(c) Receptors → Spinal cord → Sensory neuron → Motor neuron → Muscle
(d) Receptors → Sensory neuron → Spinal cord → Motor neuron → Muscle

Q11 A feature of reproduction that is common to Amoeba, Spirogyra and Yeast is that
(a) they reproduce asexually
(b) they are all unicellular
(c) they reproduce only sexually
(d) they are all multicellular

Q12 A cross between a tall plant (TT) and short pea plant (tt) resulted
in progeny that were all tall plants because
(a) tallness is the dominant trait
(b) shortness is the dominant trait
(c) tallness is the recessive trait
(d) height of pea plant is not governed by gene ‘T’ or ‘t’

Q13 What is the maximum resistance which can be made using five resistors each of 1/5 Ω?
(a) 1/5 Ω
(b) 10 Ω
(c) 5 Ω
(d) 1 Ω
Q14 A cylindrical conductor of length l and uniform area of cross- section A has resistance
R. Another conductor of length 2l and resistance R of the same material has area of cross
section
(a) A/2
(b) 3A/2
(c) 2A
(d) 3A

Q15 Choose the incorrect statement


(a) Fleming’s right-hand rule is a simple rule to know the direction of induced current
(b) The right-hand thumb rule is used to find the direction of magnetic fields due to current
carrying conductors
(c) The difference between the direct and alternating currents is that the direct current always
flows in one direction, whereas the alternating current reverses its direction periodically
(d) In India, the AC changes direction after every 1/50 SECOND

Q16 The most important safety method used for protecting home
appliances from short circuiting or overloading is
(a) earthing
(b) use of fuse
(c) use of stabilizers
(d) use of electric meter

Q17 For a current in a long straight solenoid N- and S-poles are created at the two ends.
Among the following statements, the incorrect statement is
(a) The field lines inside the solenoid are in the form of straight lines which indicates
that the magnetic field is the same at all points inside the solenoid
(b) The strong magnetic field produced inside the solenoid can be used to magnetise a piece
of magnetic material like soft iron, when placed inside the coil
(c) The pattern of the magnetic field associated with the solenoid is different from the pattern
of the magnetic field around a bar magnet
(d) The N- and S-poles exchange position when the direction of current through the solenoid
is reversed

For question numbers 18 ,19 and 20 two statements are given-one labelled Assertion (A)
and the other labelled Reason (R). Select the correct answer to these questions from the
codes (a), (b), (c)and (d) as given below :
(a) Both A and R are true and R is correct explanation of the assertion.
(b) Both A and R are true but R is not the correct explanation of the assertion.
(c) A is true but R is false.
(d) A is false but R is true.
18 Assertion (A) : Decomposition of vegetable matter into compost is an example of
exothermic reactions.
Reason (R) : Exothermic reaction are those reactions in which heat is evolved.
Q19 Assertion (A) : In anaerobic respiration, one of the end product is alcohol.
Reason (R) : There is an incomplete breakdown of glucose.
Q20 Assertion(A) : Mutation is sudden change in the genetic material.
Reason (R) : Variation is useful for the survival of species over time.
Section B
Q21 Iqbal treated a lustrous, divalent element M with sodium hydroxide. He observed the
formation of bubbles in reaction mixture. He made the same observations when this element
was treated with hydrochloric acid. Suggest how can he identify the produced gas. Write
chemical equations for both the reactions.
OR
(i) Give the constituents of baking powder
(ii) Why cake or bread swells on adding baking powder? Write chemical equation.
Q22 In the experiment “Light is essential for photosynthesis”, why does the uncovered part
of the leaf turn blue-black after putting iodine solution?
Q23 What is a tropic movement? Explain with an example.
Q24 A student sitting at the back of the classroom cannot read clearly the letters written on
the blackboard. What advice will a doctor give to her? Draw ray diagram for the correction of
this defect.
OR
The sky appears dark instead of blue to an astronaut. State its reason.
Q25 Why do fishes die when taken out of water?
Q26 Why is improper disposal of waste a curse to environment?

SECTION C
Q27 Write the balanced chemical equations for the following reactions
(a) Sodium carbonate on reaction with hydrochloric acid in equal molar concentrations
gives sodium chloride and sodium hydrogen carbonate.
(b) Sodium hydrogen-carbonate on reaction with hydrochloric acid gives sodium chloride,
water and liberates carbon dioxide.
(c) Copper sulphate on treatment with potassium iodide precipitates cuprous iodide
, liberates iodine gas and also forms potassium sulphate.
Q28 What happens when chlorine is passed over slaked lime at 313K? Write chemical
equation of the reaction involved and state two uses of the product obtained.
Q29 Describe the process of urine formation in kidneys.
OR
What would be the consequences of deficiency of hemoglobin in your body?
Q30 A convex lens of focal length 20 cm can produce a magnified virtual as well as real
image. Is this a correct statement? If yes, where shall the object be placed in each case for
obtaining these images?
Q31 Is the position of a star as seen by us its true position? Justify your answer.
Q32 What are decomposers? What will be the consequence of their absence in an ecosystem?
Q33 Why does a magnetic compass needle pointing North and South in the absence of a
nearby magnet get deflected when a bar magnet or a current carrying loop is brought near it.
Describe some salient features of magnetic lines of field concept.
OR
What is the role of fuse, used in series with any electrical appliance? Why should a fuse with
defined rating not be replaced by one with a larger rating?

SECTION D
Q34 Explain the given reactions with the examples
(a) Hydrogenation reaction
(b) Oxidation reaction
(c) Substitution reaction
(d) Saponification reaction
(e) Combustion reaction
OR
(I) Write the name and formula of the first member of the series of carbon
compounds having functional group
(II) Write the number of covalent bonds in the molecule of butane, C4H10
(III) Name the following compounds :
(a) CH3 – CH2 – OH

Q35 Distinguish between pollination and fertilisation. Mention the site and product of
fertilisation in a flower. Draw a neat, labelled diagram of a pistil showing pollen tube
growth and its entry into the ovule.
OR
(I) What happens when
(a) accidently, Planaria gets cut into many pieces-
(b) Bryophyllum leaf falls on the wet soil
(c) on maturation sporangia of Rhizopus bursts?
(II) What is the effect of DNA copying, which is not perfectly accurate, on the
reproduction process? How does the amount of DNA remain constant through
each new generation is a combination of DNA copies of two individuals?
Q36 What is meant by electric current? Name and define its SI unit. In a conductor
electrons are flowing from B to A. What is the direction of conventional current? Give
justification for your answer. A steady current of 1 ampere flows through a conductor.
Calculate the number of electrons that flows through any section of the conductor in 1
second. (Charge on electron 1.6 X 10-19 coulomb)

SECTION E
37 Read the following and answer any four questions from (i) to (v).
On the basis of reactivity of different metals with oxygen, water and acids as well as
displacement reactions, the metals have been arranged in the decreasing order of their
reactivities. This arrangement is known as activity series or reactivity series of metals.
The basis of reactivity is the tendency of metals to lose electrons. If a metal can lose electrons
easily to form positive ions, it will react readily with other substances. Therefore, it will be a
reactive metal. On the other hand, if a meal loses electrons less rapidly to form a positive ion,
it will react slowly with other substances. Therefore, such a metal will be less reactive.
(i) Which of the following metals is less reactive than hydrogen?
(a) Copper
(b) Zinc
(c) Magnesium
(d) Lead
(ii) Which of the following metals is more reactive than hydrogen?
(a) Mercury
(b) Platinum
(c) Iron
(d) Gold
(iii) Which of the following metals reacts vigorously with oxygen?
(a) Zinc
(b) Magnesium
(c) Sodium
(d) Copper
(iv) Which of the following represents the correct order of reactivity for the given
metals?
(a) Na > Mg > Al > Cu
(b) Mg > Na > Al > Cu
(c) Na > Mg > Cu > Al
(d) Mg > Al > Na > Cu
(v) Hydrogen gas is not evolved when a metal reacts with nitric acid. It is because HNO,
is a strong oxidising agent. It oxidises the H, produced to water and itself gets reduced
to any of the nitrogen oxides (N,O, NO, NO2). But _____________ and _____________
react with very dilute HNO3 to evolve H2 gas.
(a) Pb, Cu
(b) Na, K
(C) Mg, Mn
(d) Al, Zn
Q38 Read the following and answer any four questions from (i) to (v).
Pea plants can have smooth seeds or wrinkled seeds. One of the phenotypes is completely
dominant over the other. A farmer decides to pollinate one flower of a plant with smooth
seeds using pollen from plant with wrinkled seeds. The resulting pea pod has all smooth
seeds.
(I) Which of the following conclusions can be drawn?
(1) The allele for smooth seeds is dominated over that of wrinkled seeds.
(2) The plant with smooth seeds is heterozygous.
(3) The plant with wrinkled seeds is homozygous.
(a) 1 only
(b) 1 and 2 only
(c) 1 and 3 only
(d) 1, 2 and 3
(II) Which of the following crosses will give smooth and wrinkled seeds in
same proportion?
(a) RR X rr
(b) Rr x rr
(C) RRX Rr
(d) rrrr
(III) Which of the following cross can be used to determine the genotype of a
plant with dominant phenotype?
(a) RR RR
(b) Rr x Rr
(c) Rr RR
(d) RR x rr
(IV) On crossing of two heterozygous smooth seeded plants (Rr), a total of
1000 plants were obtained in F1
generation. What will be the respective number of smooth and wrinkled
seeds obtained in F1 generation?
(a) 750, 250
(b) 500, 500
(C) 800, 200
(d) 950, 50
(V) The characters which appear in the first filial generation are called
(a) recessive characters
(b) dominant characters
(c) lethal characters
(d) non-mendelian characters.
Q39 Read the following and answer any four questions from (i) to (v).
The spherical mirror forms different types of images when the object is placed at different
locations. When the image is formed on screen, the image is real and when the image does
not form on screen, the image is virtual. When the two reflected rays meet actually, the image
is real and when they appear to meet, the image is virtual. A concave mirror always forms a
real and inverted image for different positions of the object. But if the object is placed
between the focus and pole. the image formed is virtual and erect.
A convex mirror always forms a virtual, erect and diminished image. A concave mirror is
used as doctor’s head mirror to focus light on body parts like eyes, ears, nose etc., to be
examined because it can form erect and magnified image of the object. The convex mirror is
used as a rear view mirrors in automobiles because it can form an small and erect image of an
object.
(i) When an object is placed at the centre of curvature of a concave mirror, the image
formed is
(a) larger than the object
(b) smaller than the object
(c) same size as that of the object
(d) highly enlarged.

(ii) No matter how far you stand from a mirror, your image appears erect. The mirror
is likely to be
(a) plane
(b) concave
(c) convex
(d) either plane or convex
(iii) A child is standing in front of a magic mirror. She finds the image of her head
bigger, the middle portion of her body of the same size and that of the legs smaller. The
following is the order of combinations for the magic mirror from the top.
(a) Plane, convex and concave
(b) Convex, concave and plane
(c) Concave, plane and convex
(d) Convex, plane and concave
(iv) To get an image larger than the object, one can use
(a) convex mirror but not a concave mirror
(b) a concave mirror but not a convex mirror
(c) either a convex mirror or a concave mirror
(d) a plane mirror
(v) A convex mirror has wider field of view because
(a) the image formed is much smaller than the object and large number of images can be
seen.
(b) the image formed is much closer to the mirror
(c) both (a) and (b)
(d) none of these
KENDRIYA VIDYALAYA SANGATHAN, JAMMU REGION
SAMPLE PAPER SET 14
CLASS - X SUBJECT : SCIENCE (086)
TIME : 3 hours M. M : 80

General instructions:
i. All questions are compulsory.
ii. The question paper has Five sections and 39 questions. All questions are compulsory.
iii. Section–A has 20 questions of 1 mark each.
iv. Section–B has 6 questions of 2 marks each;
v. Section–C has 7 questions of 3 Marks each.
vi. Section D has 3 questions of 5 marks each.
vii. Section D has 3 CCT based questions ,4 marks each.
iv. Internal choices have been provided in some questions. A student has to attempt only one
of the alternatives in such questions.
Section A
Q1. Priyanka added a few copper pieces to 50 ml CuSO4 solution in a test tube. The correct
observation for any colour change made by her is?
A) Pale green solution turned colourless b) colourless solution turned blue
c) blue solution turned colourless d) Blue solution remained blue
Q2. Keeping the diagram in the mind, brown fumes are likely to produce after the reaction.
Name the product?

a) CO2 b) NO2
b) SO2 d) SO3

Q3. On adding a strip of Aluminium to a solution of ZnSO4. The colour of the solution
a) Remains same c) turns light green
b) Turns light blue d) turns brown
Q4. Which of the following is an exothermic process?
i) Reaction of water with quick lime
ii) Dilution of an acid
iii) Evaporation of water
iv) Sublimation of camphor
a) I and ii b) ii and iii c) i and iv d) sublimation of camphor
Q5. Name the compound which on treatment with chlorine yields Bleaching powder?
a) Slaked lime b) HCl c) potassium hydroxide d) calcium chloride
Q6. To prevent tooth decay, we are advised to brush our teeth regularly. Nature of paste used
commonly?
a) Acidic b) basic c) Neutral d) corrosive
Q7. Food cans are coated with tin and not with zinc because-
a) Zinc is costlier than tin
b) Zinc has higher melting point than tin
c) Zinc is more reactive than tin
d) Zinc is less reactive than tin
Q8. Name the functional group present in propanone?
a) Alcohol group c) Keto group
b) Carboxylic group d) none of these
Q 9 Choose the function of the pancreatic juice from the following?
a) Trypsin digests proteins and lipase carbohydrates
b) Trypsin digests emulsified fats and lipase proteins
c) Trypsin and lipase digests fats
d) Trypsin digest proteins and lipase emulsified fats.
Q10. What does A and B in the diagram depicts?

a) Single circulation c) double circulation


b) Triple circulation d) pulmonary circulation

Instruction for question number 11 to 14.


A. If both A and R are true and R is the correct explanation of A.
B. If both A and R are true and R is not the correct explanation of A
C. If A is true but R is False
D. If both A and R are false.
Q11. Assertion: Veins are thick walled as compared to Artery.
Reason: Veins distribute oxygenated blood to the whole body.
Q 12. Assertion: Planaria reproduce through Asexual method.
Reason: Asexual method used by planaria is Regeneration
Q13: Assertion: The experiences of an individual during its lifetime cannot be passed on its
progeny.
Reason: Change in non- reproductive tissues cannot be passed on to DNA of germ cells.
Q14. Assertion: The pattern of magnetic field lines around an solenoid is similar to the
magnetic field around a bar magnet.
Reason: The magnetic field is the same at all the points inside a solenoid.
Q15.
A compass is to be placed near a bar magnet with unknown poles. Outside the magnetic field,
the compass needle is pointing towards North as shown below:

Q 16. Which of the following is not associated with growth of plants?


a) Auxin c) Gibberellins
b) Cytokinin d) Abscisic acid
Q 17. Two pink coloured flowers on crossing resulted in 1 red, 2 pink, and 1 white flower
progeny. The nature of the cross will be
a) Double fertilization
b) Self-pollination
c) Cross fertilization
d) No fertilization
Q 18. The electrical resistivity depends upon –
a) Its length c) its thickness
b) Its shape d) nature of the material
Q 19. An in candescent lamp of resistance 80 ohms draws a current of .75 A. Find the
Line voltage?
a) 60 v b) 80 v c) 100 v d) 120 v
Q20. The amount of work done in joules when 1unit electric charge moves from one point to
another point in an electric circuit is called :
a) Electric Current
b) Electric Resistance
c) Electric Conductance
d) Potential Difference
Section B
Q 21. What are amphoteric oxides? Give examples?
Q 22. How are fats digested in our bodies? Where does this process takes place?
Or
(i) Name two waste products which are stored in old xylem in plants.
(ii) Name the process by which plants get rid of excess water. Name the pores through which
this process takes place.

Q 23. Study the given diagram: Name the parts ‘A’ and ‘B’ and state one function of each.

Q 24. What constitutes the central nervous system and peripheral nervous system?
Q 25. What is astigmatism? How it can be corrected?
Q 26. Why should bio-degradable and non-biodegradable waste discarded into separate dust
bins?
Section C
Q 27.A silver article generally turns black when kept open for a few day. A black film is
produced over it. Give reasons why?
Q28. In a gas jar containing hydrogen chloride gas, a dry blue litmus paper is dropped. What
change is observed? Now a blue litmus paper is moistened and dropped into gas jar. State
what is observed and give a reason for this?
Q29. (i) Why are Artery being thick walled?
(ii) What is A.T.P.? How is it formed?
Q30. An object is placed between infinity and the pole of a convex mirror. Draw a ray
diagram and state the position, the relative size and the nature of the image formed. (AI
Q31. On what factors does the magnetic field produced by a current carrying solenoid
depend?
Q32. What is being depicted in the below mentioned scheme. Explain?

ii) What will happen if we kill all the organisms in one trophic level?
Q 33. Why does the clear sky appear blue? How would the sky appear in the absence of
earth’s atmosphere?
Or
Below is the ray diagram to show the refraction of light through a glass prism. Mark on it (a)
the incident ray, (b) the emergent ray and (c) the angle of deviation d) angle of incidence
a) Angle of prism

SECTION D
Q 34: Describe the role of prostate gland, seminal vesicle, and testes in the human male
reproductive system.
ii) How is surgical removal of unwanted pregnancies misused?
iii) Explain the role of oral contraceptive pills in preventing contraception.
Q35. Write the structural formula of ethanol. What happens when it is heated with excess of
concentrated H2SO4. Write the chemical equation for the reaction stating the role of
concentrated sulphuric acid in this reaction?
Q36.
List the advantages of connecting electrical devices in parallel with an electrical source
instead of connecting them is series.
ii) What would be the total value of resistance in the below shown diagram?

A)

B)

SECTION D
Q 37. The elements have been classified into metals, non-metals, and metalloids. Metals are
generally hard, good conductor of heat and electricity, malleable, ductile and have shining
lustre. The non-metals are soft, generally, poor conductors of heat and electricity, brittle and
do not possess lustre like metals. The metalloids show characteristics of metals as well as
non- metals. Most of the elements in the periodic table are metals. Metals also form alloys
which are homogenous mixtures and whose properties are in between those of constituents.

Q 1. What is the nature of the oxides of metals?


a) Acidic b) basic c) amphoteric d) none
Q2. Give one example of non-metal which is a good conductor of electricity?
a) aluminium b) graphite c) diamond d) copper
Q3. Define metalloid?

Q 38. The two sexes participating in sexual reproduction must be somewhat different from
each other for a number of reasons. How is the sex of a new born individual determined?
Different species use very different strategies for this. Some rely entirely on environmental
cues. In human beings, the sex of the individual is largely genetically determined. In other
words, the genes inherited from our parents decide whether we will be boys or girls.

Q1: How many chromosomes are there in human beings?

Q2: A women has only daughters. Analyse the situation genetically and provide a suitable
explanation?

Q3: The gene contains chemically

A) DNA B) RNA C) Chromosome d) Both DNA and RNA

Q 39. In our home, we receive supply of electric power through a main supply supported
through overhead electric poles or by underground cables. One of the wires in the supply,
usually with red insulation cover, is called Live wire (positive). Another wire, with black
insulation, is called neutral wire (or negative). In our country, the potential difference
between the two is 220 V.

Q1. What does the wire with green insulation represents?

a) Fuse b) earth wire c) live wire d) neutral wire


Q2. What is the value of domestic electric potential difference in Indian house hold?

a) 20 V B) 220 V C) 300 V D) 120 VW

Q3. What is short- circuiting?

Q4. How can we avoid overloading in electric circuit?


KENDRIYA VIDYALAYA SANGATHAN, JAMMU REGION
SAMPLE PAPER SET 15
CLASS - X SUBJECT : SCIENCE (086)
TIME : 3 hours M. M : 80

General Instructions:
a) This question paper consists of 39 questions in 5 sections.
b) All questions are compulsory. However, an internal choice is provided in some
questions. A student is expected to attempt only one of these questions.
c) Section A consists of 20 objective type questions carrying 1 mark each.
d) Section B consists of 6 Very Short questions carrying 02 marks each. Answers to
these questions should in the range of 30 to 50 words.
e) Section C consists of 7 Short Answer type questions carrying 03 marks each. Answers
to these questions should in the range of 50 to 80 words
f) Section D consists of 3 Long Answer type questions carrying 05 marks each. Answer
to these questions should be in the range of 80 to 120 words.
g) Section E consists of 3 source-based/case-based units of assessment of 04 marks each
with sub-parts

Q.1 Which among the following is (are) double displacement reaction(s)? 1

(t) Pb + CuCl → PbCl2 + Cu

(ii) Na2SO4 + BaCl2 → BaSO4 +2NaCl

(iii) C +O2 → CO2

(iv) CH4 + 2O2 → CO2 +2H2O

(a) (i) and(iv)

(b) (ii)only

(c) (i) and(ii)

(d) (iii) and(iv)

Q.2 Which of the following is correct balanced equation:- 1

a) Fe +H₂O→ Fe₃O₄ +H₂↑

b)2Fe+4H₂O→Fe₃O₄+4H₂↑

c) 3Fe+4H₂O→Fe₃O₄+4H₂↑
d) 3Fe+ H₂O→Fe₃O₄+ H₂↑

Q.3 MnO2 + 4HCl → MnCl2 + H2O + Cl2 The oxidising agent is 1


(a) MnO2
(b) HCl
(c) MnCl2
(d) Ag + Zn(NO3)3

Q.4 Brine is an 1
(a) aqueous solution of sodium hydroxide
(b) aqueous solution of sodium carbonate
(c) aqueous solution of sodium chloride
(d) aqueous solution of sodium bicarbonate

Q.5 1

The apparatus given in the adjoining figure was set up to demonstrate electrical
conductivity.

Which of the following statement(s) is (are) correct?


(i) Bulb will not glow because electrolyte is not acidic.

ii)Bulb will glow because HCl is a strong acid and furnishes ions for conduction.

(iii) Bulb will not glow because circuit is incomplete.


(iv) Bulb will not glow because it depends upon the type of electrolytic solution.
(a) (i) and (iii)
(b) (ii) and (iv)
(c) (ii) only
(d) (iv) only
Ionic compounds have: -
Q.6 (a) High melting and boiling points and conduct electricity in solution. 1
(b) High melting and low boiling points and doesn’t conduct electricity in solution.
(c) Low melting and high boiling points and doesn’t conduct electricity in solution.
(d) Low melting and boiling points and conduct electricity in solution.

Q.7 1

In the experiment given here water will rise in the tube because
(a) Oxygen of air in the flask will be taken up by the germinating seeds
(b) Carbon dioxide given out by the germinating seeds will be absorbed by KOH.
(c) Carbon dioxide given out we'll go through the glass tube and push water up into the
tube
(d) Moisture in the germinating seeds will reach the water in the beaker through the
delivery tube.
The correct reason of water to rise in the tube is
(i) (a)
(ii) (b)
(iii) (c)
(iv) (d)

Which of the following statement(s) is (are)true about the heart?


Q.8 1
(i) The left atrium receives oxygenated blood from different parts of the body while the
right atrium
receives deoxygenated blood from lungs.
(ii) Left ventricle pumps oxygenated blood to different body parts while right ventricle
pumps
deoxygenated blood to lungs.
(iii) Left atrium transfers oxygenated blood to the right ventricle which sends it to
different body parts.
(iv) The right atrium receives deoxygenated blood from different parts of the body while
the left
ventricle pumps oxygenated blood to different parts of the body.

What is the correct direction of flow of electrical impulses?


Q.9 1

Q.10 The correct sequence of reproductive stages seen in flowering plants is 1


(а) gametes, zygote, embryo, seedling
(b) zygote, gametes, embryo, seedling
(c) seedling, embryo, zygote, gametes
(d) gametes, embryo, zygote, seedling
Q.11 A man with blood group A marries a woman having blood group O. What will be the 1
blood group of the child?
(a) O only
(b) A only
(c) AB
(d) A, B, AB and O only

Q.12 Pure-bred pea plant A is crossed with purebred pea plant B. It is found that the plants 1
which look like A do not appear in F1 generation but re-emerge in F2 generation. Which
of the plants A and B are tall and dwarf?
(a) A are tall and B are dwarf.
(b) A are tall and B are also tall.
(c) A are dwarf and B are also dwarf
(d) A are dwarf and B are tall
The image shows a combination of 4 resistors.
Q.13 1

What is the net resistance between the two points in the circuit?
(a) 0.5 Ω
(b) 1.0 Ω
(c) 1.5 Ω
(d) 2.0 Ω

What is the relationship between resistance and current?


Q.14 1
(a) They are directly related to each other.
(b) They are inversely related to each other.
(c) The resistance has a greater magnitude than current.
(d) The current has a greater magnitude than resistance.

The strength of magnetic field inside a long current-carrying straight solenoid is:
Q.15 a) More at the ends then at the center 1
b) Minimum in the middle
c) Same at all points
d) Found to increase from one end to the another

The factors on which one magnetic field strength produced by current carrying solenoids
Q.16 depends are 1
(a) Magnitude of current
(b) Number of turns
(c) Nature of core material
(d) All of the above

DIRECTIONS for the question 17 to 20: In each of the questions given below, there are
two statements marked as Assertion (A) and Reason (R). Mark your answer as per the
codes provided below:

1. Both A and R are true and R is the correct explanation of A.


2. Both A and R are true but R is not the correct explanation of A.
3. A is true but R is false.
4. A is false but R is true.
5. Both A and R are false.

Assertion(A): Silver Bromide is used in black and white photography.


Q.17 1
Reason(R): Silver Bromide decomposes in the presence of sunlight

Assertion(A): Genes present in every cell of an organism control the traits of the
Q.18 1
organisms.
Reason (R): Gene is specific segment of DNA occupying specific position on a
chromosome

Assertion: The accumulation of lactic acid in the muscles causes muscle cramps.
Q.19 1
Reason: During vigorous physical exercise leg muscles respire anaerobically.

Assertion(A): A compass needle is placed near a current carrying wire. The deflection of
Q.20 1
the compass needle decreases when the compass needle is displaced away from the wire.
Reason (R): Strength of a magnetic field decreases as one moves away from a current
carrying conductor.

SECTION -B

A metal ‘X’ combines with a non-metal ‘Y’ by the transfer of electrons to form a
Q.21 2
compound Z.
(i) State the type of bond in compound Z.
(ii) What can you say about the melting point and boiling point of compound Z?

Or

Give reason for the following:


(i) Hydrogen gas is not evolved when most of the metals react with nitric acid.
(ii) Zinc oxide is considered as an amphoteric oxide

Name the glands present in the wall of the stomach that release secretions for digestion of
Q.22 2
food. Write the three components of secretion that are released by these glands.

Why do herbivores have longer, small intestine than carnivores?


Q.23 2
Name one plant hormone which inhibits growth. Write its one more function.
Q.24 2
A student is unable to see clearly the words written on the blackboard placed at a distance
Q.25 2
of approximately 4 m from him. Name the defect of vision the boy is suffering from.
Explain the method of correcting this defect.

Or

A narrow PQ of white light is passing through a glass prism ABC as shown in the
diagram. Trace it on your answer sheet and show the path of the emergent beam as
observed on the screen DE.

(i) Write the name and cause of the phenomenon observed.


(ii) Where else in nature is this phenomenon observed?

Explain phenomenon of “biological magnification”. How does it affect organisms


Q.26 2
belonging to different trophic levels particularly the tertiary consumers?

SECTION-C
State the type of chemical reactions, represented by the following equations: (Board Term
Q.27 3
I, 2014)
(a) A + BC → AC + B
(b) A + B → C
(c) PQ + RS → PS + RQ

A white powder is added while baking breads and cakes to make them soft and fluffy.
Q.28 3
Write the name of the powder. Name its main ingredients. Explain the function of each
ingredient. Write the chemical reaction taking place when the powder is heated during
baking

Write different ways in which glucose is oxidised to provide energy in human body.
Q.29 3
Write the products formed in each case.

Or

Describe the structure and function of nephron with the help of diagram.

A spherical mirror produces an image of magnification -1 on a screen placed at a distance


Q.30 3
of 50 cm from the mirror.
(a) Write the type of mirror.
(b) Find the distance of the image from the object.
(c) What is the focal length of the mirror?

Draw a labelled diagram to explain the formation of a rainbow in the sky.


Q.31 3
Current-time graphs from two different sources are shown in the following diagrams.
Q.32 3

Now answer the following questions.

1. Name the type of current shown by graph A and graph B.


2. Name any one source of the current shown by graph A and graph B.
3. What is the frequency of current in case B?

Or

Explain the effect on the magnetic field produced at a point in a current carrying circular
coil due to:
(i) increase in the amount of current flowing through it
(ii) increase in the distance of point from the coil
(iii) increase in the number of turns of the coil.

(a) Construct a terrestrial food chain comprising four trophic levels.


Q.33 (b) What will happen if we kill all organisms in one trophic level? 3
(c) Calculate the amount of energy available to the organisms at the fourth trophic level.
If the energy available to the organisms at the second trophic level is 2000 J

SECTION -D

A compound X is formed by the reaction of a carboxylic acid C2H4O2 and an alcohol in


Q.34 presence of a few drops of H2SO4. The alcohol on oxidation with alkaline 5
KMnO4 followed by acidification gives the same carboxylic acid as used in this reaction.
Give the names and structures of (a) carboxylic acid, (b) alcohol and (c) the compound X.
Also write the reaction.

Or

A salt X is formed and a gas is evolved when ethanoic acid reacts with sodium hydrogen
carbonate. Name the salt X and the gas evolved. Describe an activity and draw the
diagram of the apparatus to prove that the evolved gas is the one which you have named.
Also, write chemical equation of the reaction involved.

a) What is Pollination? Give its two types.


Q.35 b) Draw a longitudinal section of female reproductive part of a flower showing 5
germination of pollen grain. Label on it the following :
i)Stigma
ii)Pollen tube with a male germ cell
iii) Female germ cell
OR

i)Draw a sectional view of human female reproductive system and label the following
parts:
Where the development of egg occurs
Where fertilization takes place
Where the fertilized egg gets implanted
ii) Describe the changes the uterus undergoes
a) To receive the zygote
b) If zygote is not formed

What is solenoid? Draw the pattern of magnetic field lines of (i) a current carrying
Q.36 solenoid(ii) A bar magnet. List two distinguishing features between the two field. 5

SECTION -E

Iqbal treated a lustrous, divalent element M with sodium hydroxide. He observed


Q.37 the formation of bubbles in the reaction mixture. He made the same observations
when this element was treated with hydrochloric acid.

(a) Suggest how can he identify the produced gas.


(b) Write chemical equations for both the reactions.
Or
Anil and his neighbour Sunil had got their garden fenced with iron rods. Next day
Anil saw that Sunil was painting the iron fence. Sunil suggested Anil to do the
same to increase the longevity of the iron rods by preventing corrosion. Anil
argued that it is a waste of time and his iron rods were strong enough. After
reading the above passage, answer the following questions.

(i) Whose opinion was correct? Justify.


(ii) Mention two methods (other than painting) to prevent iron from corrosion.
(iii) What is the chemical formula of rust?
Pea plants can have smooth seeds or wrinkled seeds. One of the phenotypes is completely
Q.38 dominant over the other. A farmer decides to pollinate one flower of a plant with smooth 4
seeds using pollen from plant with wrinkled seeds. The resulting pea pod has all smooth
seeds.

(i) Which of the following conclusions can be drawn?


(1) The allele for smooth seeds is dominated over that of wrinkled seeds.
(2) The plant with smooth seeds is heterozygous.
(3) The plant with wrinkled seeds is homozygous.
(a) 1 only
(b) 1 and 2 only
(c) 1 and 3 only
(d) 1, 2 and 3
(ii) Which of the following crosses will give smooth and wrinkled seeds in same
proportion?
(a) RR X rr
(b) Rr x rr
(C) RRX Rr
(d) rrrr

(iii) Which of the following cross can be used to determine the genotype of a plant with
dominant phenotype?
(a) RR RR
(b) Rr x Rr
(c) Rr RR
(d) RR x rr

(iv) On crossing of two heterozygous smooth seeded plants (Rr), a total of 1000 plants
were obtained in F1
generation. What will be the respective number of smooth and wrinkled seeds obtained in
F1 generation?
(a) 750, 250
(b) 500, 500
(C) 800, 200
(d) 950, 50

The lenses form different types of images when object placed at different locations. When
Q.39 a ray is incident parallel to the principal axis, then after refraction, it passes through the 4
focus or appears to come from the focus.

When a ray goes through the optical centre of the lens, it passes without any deviation. If
the object is placed between focus and optical centre of the convex lens, erect and
magnified image is formed.

As the object is brought closer to the convex lens from infinity to focus, the image moves
away from the convex lens from focus to infinity. Also, the size of image goes on increasing
and the image is always real and inverted.

A concave lens always gives a virtual, erect and diminished image irrespective to the
position of the object.
(i)The location of image formed by a convex lens when the object is placed is

(a)At infinity
(b)At 2F
(c)At optical centre
(d)Between F and 2F

(ii) When the object is placed at the focus of concave lens, the image formed is
(a) real and smaller
(b) virtual and inverted
(c) virtual and smaller
(d) real and erect

(iii) The size of image formed by a convex lens when the object is placed at the focus of
convex lens is

(a) Small
(b) Point in size
(c) Highly magnified
(d) Same as that of object
(iv) When the object is placed at 2F in front of convex lens, the location of image is
(a) at F
(b) at 2 F on the other side
(c) at infinity
(d) between F and optical centre

You might also like